Small Bowel Flashcards

1
Q

A 55-year-old female with a long standing history of Crohn’s disease presents to the ER with severe diarrhea. She has a history of multiple bowel resections and is frequently admitted to the hospital for rehydration and electrolyte repletion. All of the following are true of her condition EXCEPT:

A. Increased likelihood of gallstones

B. increased likelihood of kidney stones

C. Mainstay of treatment is TPN and enteral feeding when tolerated

D. Diagnosis is made if patient has less than 150cm of small bowel or less than 70cm with a competent ileocecal valve

E. Jejunal resections are better tolerated than ileal resections

A

D. Diagnosis is made if patient has less than 150cm of small bowel or less than 70cm with a competent ileocecal valve

How well did you know this?
1
Not at all
2
3
4
5
Perfectly
2
Q

A 42-year-old male has an extensive history of partial small bowel obstructions that resolve with conservative management. He has not had any previous surgical interventions. After thorough workup, the patient has a Tx-pertechnetate scan that shows uptake proximal to the ileocecal valve. What is the BEST next step in management?

A. Observation and await resolution of obstruction

B. Diverticulectomy when obstruction resolves

C. Diverticulectomy if patient has GI bleeding

D. Segmental resection

E. Barium UGI with small bowel follow-through

A

B. Diverticulectomy when obstruction resolves

How well did you know this?
1
Not at all
2
3
4
5
Perfectly
3
Q

Where is the largest number of hormone-producing cells found in the body?

A. The pituitary

B. The small intestine

C. The pancreas

D. The liver

A

Answer: B

The small intestine is the body’s largest reservoir of hormone-producing cells.

Multiple specialized cells within the intestinal mucosa respond to luminal stimuli and secrete over 30 peptide hormones which regulate the functions of the intestine, other organs in the gastro-entero-pancreato-biliary system, the heart, and the brain.

(See Schwartz 10th ed., p. 1145.)

How well did you know this?
1
Not at all
2
3
4
5
Perfectly
4
Q

Which of the following features is characteristic of the ileum, as opposed to the jejunum?

A. The presence of valvulae conniventes

B. The presence of Peyer patches

C. Larger vasa recta

D. Less fatty mesentery

A

Answer: B

The entire small intestine contains valvulae conniventes, also known as plicae circularis.

The jejunum has larger vasa recta, a larger diameter, and a less fatty mesentery.

The ileum contains prominent lymphoid follicles called Peyer patches.

(See Schwartz 10th ed., p. 1138.)

How well did you know this?
1
Not at all
2
3
4
5
Perfectly
5
Q

Within the intestine, epithelial cells originate rom stem cells, proliferate in the crypts, and migrate up the villus in 2 to 5 days.

This process replaces cells that are removed due to apoptosis or exfoliation.

This rapid turnover makes the small intestine susceptible to

A. Radiation damage

B. Starvation

C. Exogenous steroids

D. Hypothermia

A

Answer: A

The high cellular turnover rate of enterocytes makes the small intestine susceptible to damage by inhibitors of proliferation such as radiation and cytotoxic chemotherapy.

(See Schwartz 10th ed., p. 1138.)

How well did you know this?
1
Not at all
2
3
4
5
Perfectly
6
Q

A pocket- or sock-like outpouching on the anti-mesenteric side of the distal ileum, called a Meckel diverticulum, is caused by

A. Excessive traction on the intestine during childbirth.

B. Increased intraluminal pressure.

C. A persistent vitelline duct.

D. A mutation of the c-Mec gene.

A

Answer: C

The embryonic gut communicates with the yolk sac by mean of the vitelline duct.

Failure of this structure to obliterate by the end of gestation can result in a Meckel diverticulum.

(See Schwartz 10th ed., p. 1139.)

How well did you know this?
1
Not at all
2
3
4
5
Perfectly
7
Q

How much fluid normally enters the adult small intestine each day?

A. 2L

B. 4L

C. 6L

D. 8L

A

Answer: D

Eight to nine liters of fluid enters the small intestine daily, of which over 80% is absorbed.

This includes 2L from oral intake, 1.5L of saliva, 2.5L of gastric juice, 1.5L of bilio-pancreatic secretions, and 1L of fluid secreted by the small intestine.

(See Schwartz 10th ed., p. 1140.)

How well did you know this?
1
Not at all
2
3
4
5
Perfectly
8
Q

How are the digestion products of carbohydrates, such as glucose, galactose, and fructose, absorbed through the intestine?

A. By passive diffusion across enterocyte plasma membranes.

B. By facilitated diffusion via specific transporters such as sodium-glucose co-transporter 1 (SGLT1), glucose transporter 2 (GLUT2), and glucose transporter 5 (GLUT5).

C. By endocytosis of enterocytes on the villus.

D. By facilitated diffusion through tight junctions between enterocytes.

A

Answer: B

The three terminal products of carbohydrate digestion are transported through the enterocyte brush border membrane via facilitative transporter proteins such as the sodium-glucose cotransporter 1 (SGLT1), glucose transporter 2 (GLUT2), and glucose transporter 5 (GLUT5).

There is evidence of overexpression of these transporters, particularly SGLT1, in diabetes and obesity, and new therapeutic approaches or these conditions are designed to inhibit these transporters.

(See Schwartz 10th ed., p. 1141.)

How well did you know this?
1
Not at all
2
3
4
5
Perfectly
9
Q

A 45-year-old female with a history of Crohn’s disease presents to the ER with abdominal pain, vomiting and distension. A CT scan is performed and shows inflammatory changes and a stricture located at the proximal duodenum causing an obstruction. After one week of conservative management, the patient has no improvement in symptoms. What is the BEST surgical option for the patient?

A. No surgery unless complicated by perforation, fistula or abscess to avoid bowel shortening

B. Heineke-Mikulicz strictureplasty

C. Finney strictureplasty

D. Gastrojejunostomy

E. Whipple procedure

A

D. Gastrojejunostomy

How well did you know this?
1
Not at all
2
3
4
5
Perfectly
10
Q

A 55-year-old female with a history of Crohn’s disease presents to the ER with severe diarrhea. She has a history of multiple bowel resections and is frequently admitted to the hospital for rehydration and electrolyte repletion. All of the following are true of her condition EXCEPT:

A. Increased likelihood of gallstones

B. Increased likelihood of kidney stones

C. Mainstay of treatment is TPN and enteral feeding when tolerated

D. Diagnosis is made if patient has less than 150cm of small bowel or less than 70cm with a competent ileocecal valve

E. Jejunal resections are better tolerated than ileal resections

A

D. Diagnosis is made if patient has less than 150cm of small bowel or less than 70cm with a competent ileocecal valve

How well did you know this?
1
Not at all
2
3
4
5
Perfectly
11
Q

A 64-year-old male presents to the ER with a one week history of abdominal pain. An ultrasound shows evidence of cholecystitis. The patient improves with IV antibiotics and the decision if made to bring the patient back in one month for cholecystectomy, after the inflammation has resolved. Just prior to follow-up, the patient returns to the ER with abdominal pain and distension. He has not had a bowel movement in five days. What is the next BEST study?

A. Abdominal xray

B. CT scan abdomen and pelvis

C. Ultrasound

D. HIDA scan

E. ERCP

A

A. Abdominal xray

How well did you know this?
1
Not at all
2
3
4
5
Perfectly
12
Q

A 64-year-old male presents to the ER with a one week history of abdominal pain. An ultrasound shows evidence of cholecystitis. The patient improves with IV antibiotics and the decision is made to bring back the patient in one month for cholecystectomy, after the inflammation has resolved. Just prior to follow up, the patient returns to the ER with abdominal pain and distension. He has not had a bowel movement in five days. The findings on imaging suggest an obstruction with the appearance of a gallstone near the terminal ileum. What is the BEST management of this patient’s condition?

A. Open cholecystectomy

B. Laparotomy, proximal enterotomy, and stone removal

C. Laparotomy, terminal ileotomy, stone removal and cholecystostomy tube

D. Laparotomy, proximal enterotomy, stone removal and cholecystectomy

E. Laparotomy, terminal ileotomy, stone removal, and cholecystectomy

A

D. Laparotomy, proximal enterotomy, stone removal and cholecystectomy

How well did you know this?
1
Not at all
2
3
4
5
Perfectly
13
Q

A 50-year-old female comes to the office complaining of intermittent abdominal pain, diarrhea, and flushing of her face and neck. She also notes having recent asthmatic attacks which she never had before. A CT scan is performed that shows a non-obstructing mass in the ileum and several lesions in the liver suggestive of metastasis. What hormone is associated with this disease?

A. Histamine

B. Dopamine

C. Acetylcholine

D. Norepinephrine

E. Serotonin

A

E. Serotonin

How well did you know this?
1
Not at all
2
3
4
5
Perfectly
14
Q

A 65-year-old female is diagnosed with colon cancer in the cecum and undergoes an open right colectomy. What is the appropriate order for the return of bowel function after abdominal surgery?

A. Stomach, small bowel, colon

B. Small bowel, stomach, colon

C. Stomach, colon, small bowel

D. Small bowel, colon, stomach

E. Colon, small bowel, stomach

A

B. Small bowel, stomach, colon

*if carcinoid syndrome: small bowel

How well did you know this?
1
Not at all
2
3
4
5
Perfectly
15
Q

A newly diagnosed Crohn’s patient comes to the hospital complaining of abdominal pain. CT scan of the abdomen reveals a single 2cm area of stricture in the distal small bowel. You take the patient to the operating room and note a stricture without evidence of surrounding acute inflammation or phlegmon. The BEST treatment option is:

A. Limited resection

B. Balloon dilatation

C. Heineke-Mikulicz Stricureplasty

D. Medical management

E. Finney Strictureplasty

A

C. Heineke-Mikulicz Stricureplasty

How well did you know this?
1
Not at all
2
3
4
5
Perfectly
16
Q

A 61-year-old female sustains an iatrogenic enterotomy during abdominal laparotomy. Which layer of the intestinal wall is MOST important in maintaining tensile strength while constructing a hand-sewn anastomosis?

A. mucosa

B. submucosa

C. muscularis

D. serosa

A

B. submucosa

How well did you know this?
1
Not at all
2
3
4
5
Perfectly
17
Q

A 25-year-old females comes to your office complaining of crampy abdominal pain and diarrhea over the last few weeks. She has a colonoscopy performed that shows evidence of Crohn’s disease. All of the following favor a diagnosis of Crohn’s disease over ulcerative colitis EXCEPT?

A. Granulomas

B. Rectal involvement

C. Cobblestone appearance

D. Transmural involvement

E. Patchy areas of diseased bowel

A

B. Rectal involvement

How well did you know this?
1
Not at all
2
3
4
5
Perfectly
18
Q

A patient presents to your office with flushing, diarrhea, hepatomegaly, pulmonary stenosis, and asthma symptoms. He undergoes a CT scan of the chest, abdomen and pelvis to determine the location of the primary tumor. What is the most likely site for the primary tumor in this patient?

A. Small bowel

B. Appendix

C. Lung

D. Rectum

E. Ovary

A

A. Small bowel

How well did you know this?
1
Not at all
2
3
4
5
Perfectly
19
Q

A patient has a small bowel resection for lesion identified on CT scan. The pathology report indicates the lesion is consistent with metastatic diseases. Which of the following is the MOST likely location of the primary tumor?

A. Adrenal gland

B. Liver

C. Stomach

D. Skin

E. Thyroid

A

D. Skin

*Melanoma–> mets to small bowel

How well did you know this?
1
Not at all
2
3
4
5
Perfectly
20
Q

A 25-year-old male comes in with high-output enter-cutaneous fistula. Which of the following acid-base derangements is most likely present?

A. Metabolic acidosis

B. Metabolic alkalosis

C. Mixed respiratory and metabolic alkalosis

D. Mixed respiratory and metabolic acidosis

A

A. Metabolic acidosis

How well did you know this?
1
Not at all
2
3
4
5
Perfectly
21
Q

A 30 year- old frail lady presents to the emergency room for progressive weakness and weight loss. She just underwent right hemicolectomy for obstructing GITB 3 weeks ago. Which of the following is not a metabolic consequence of the patient’s previous surgery?

A. Megaloblastic anemia

B. Cholera-like diarrhea

C. Low levels of serum iron

D. Vitamin B12 deficiency

A

C. Low levels of serum iron

How well did you know this?
1
Not at all
2
3
4
5
Perfectly
22
Q

45/M consults for progressive abdominal dissension and constipation. Last bowel movement was 3 days ago with note of passage of gas a few hours back. Patient denies any previous abdominal surgery and any co-morbidity. Which diagnostic exam would be best to determine course of management?

A. Plain abdominal x-ray, supine and upright

B. Upper gastrointestinal series with small bowel follow-through

C. CT scan with contrast

D. MRI of abdomen

A

C. CT scan with contrast

How well did you know this?
1
Not at all
2
3
4
5
Perfectly
23
Q

A Meckel diverticulum is derived from which of the following structures?

A. Allantois

B. Connecting stalk

C. Vitelline duct

D. Urachus

A

C. Vitelline duct

How well did you know this?
1
Not at all
2
3
4
5
Perfectly
24
Q

A 32-year-old male presents with high grade fever of 2 weeks duration, lower abdominal pain and tenderness of 2 days duration. PE shows absent bowel sounds, abdominal rigidity and rebound tenderness. Their most likely differential diagnosis is?

A. Ileocecal tuberculosis

B. Meckel diverticulitis

C. Ruptured appendicitis

D. Typhoid ileitis with perforation

A

D. Typhoid ileitis with perforation

How well did you know this?
1
Not at all
2
3
4
5
Perfectly
25
Q

The small intestine typically reabsorbs what percentage of the fluid that passes through its lumen?

A. 10 
B. 20 
C. 40 
D. 50 
E. 80
A

ANSWER: E

COMMENTS: The intestine has a remarkable ability to absorb and secrete large quantities of fluid.

Absorption of water is a net result of fluxes into and out of the intestinal lumen.

An average person consumes approximately 1 to 1.5 liters (L) of water per day.

The gastrointestinal tract (GI) secretes an additional 5 to 10 L, including 1 to 2 L of saliva, 2 to 3 L of gastric secretions, 0.5 L of biliary secretions, 1 to 2 L of pancreatic juice, and 1 L of intestinal secretions.

The small intestine reabsorbs nearly 80% of the fluid that passes through it.

A rapid bidirectional movement of fluid in the intestinal lumen accomplishes this dynamic process.

A total of 6 to 11 L of water enters the duodenum every day, but only 1 to 1.5 L arrives in the colon.

Alterations in this fine balance caused by either impaired absorption or augmented secretion can result in overall net secretion of water and result in diarrhea.

How well did you know this?
1
Not at all
2
3
4
5
Perfectly
26
Q

With regard to ileostomy physiology, which of the following statements is true?

A. Daily output from an established ileostomy is approximately 1500 mL.

B. Ileostomy output can increase by 50% at times of dietary indiscretion.

C. With dehydration, the concentration of sodium output from the ileostomy rises.

D. Compared with normal ileal fluid, ileostomy effluent contains a 100-fold increase in aerobes and a 2500-fold increase in coliform bacteria.

E. The microbiologic flora of ileostomy output is similar to that of normal ileal fluid.

A

ANSWER: D

COMMENTS: The daily output from an established ileostomy is typically 500 to 1000 mL. Although there is a great deal of variation in daily output among individuals, the output in a given patient varies only about 20% with changes in diet or with episodes of gastroenteritis.

The usual ileostomy sodium concentration is 115 mEq/L, although the concentration rises and falls with changes in total body sodium.

With dehydration, the sodium concentration falls and the potassium level rises as a result of the ability of the terminal ileum to conserve sodium in times of salt depletion.

Normally, the sodium-to-potassium ratio is about 12:1.

The microbiologic flora of ileostomy output is markedly different from that of normal ileal fluid.

The total number of bacteria is 80 times greater, and there is a 100-fold increase in the number of aerobes, a 2500- fold increase in the number of coliform bacteria, and an increase in the total number of anaerobes.

How well did you know this?
1
Not at all
2
3
4
5
Perfectly
27
Q

A patient undergoes a contrast radiograph of the small bowel for evaluation of intestinal pseudo-obstruction. The average transit time from the duodenum to the cecum is:

A. 30 min 
B. 1 h
C. 3 h
D. 5 h
E. 7 h
A

ANSWER: B

COMMENTS: In healthy humans, the transit time of barium contrast from the duodenum to the terminal ileum varies greatly, from 30 min to 5 h.

The average (>80%) transit time is approximately 1 h.

However, the composition of the meal affects the rate of occur- rence and propagation of contractions during the postprandial period.

The frequency of contraction is greatest with meals containing glucose and least after meals high in fat.

Therefore transit is regulated to optimize absorption of nutrients.

How well did you know this?
1
Not at all
2
3
4
5
Perfectly
28
Q

Which of the following statements about small bowel motility is true?

A. Oral feeding stimulates the production of migrating motorcomplexes (MMCs).

B. If motility is impaired, absorption of nutrients is similarly affected.

C. MMCs are peristaltic contractions occurring at 10- to 20-min intervals.

D. Vagotomy-induced diarrhea is the result of increased
secretion secondary to denervation.

E. Segmental bowel resection causes a temporary interruption of MMCs, but the clinical results are usually insignificant.

A

ANSWER: E

COMMENTS: MMCs are sets of propagated aboral peristaltic contractions occurring at 90-min intervals during fasting. They are one of the most recognizable and reproducible GI motility activities.

The activity fronts of MMCs usually originate high in the stomach, propagate distally, and end in the ileum, usually at the mid-ileal level.

Oral feeding inhibits MMCs and results in irregular, nonpropagating contractions throughout most of the small intestine.

This postprandial inhibition generally persists for 3 to 4 h after a meal and is most pronounced with lipids.

Although this motility pattern appears to be disorganized, there is a distal progression of chyme.

Absorption is not affected by intestinal motility. Enteral feedings can therefore be used safely and efficiently in postoperative patients in whom motility may be altered.

Both gastric and small bowel motility can be affected by exogenous conditions.

The small bowel is less sensitive than the stomach to general anesthesia and laparotomy, each of which decreases the frequency of MMCs.

The frequency of MMCs returns to normal within 6 to 24 h in the absence of peritonitis or abscess formation.

The tone of the stomach is affected more than that of the small bowel by general anesthesia and laparotomy and may take longer than 24 h to normalize.

This may explain the occurrence of postoperative nausea and emesis.

Vagotomy-induced diarrhea is a result of persistence of the sustained, organized wave of MMCs during the postprandial state due to loss of vagal parasympathetic inhibition of MMCs.

Segmental small bowel resection or denervation temporarily reduces the frequency of MMCs with a resultant temporary impairment of motility.
Resection or denervation does not produce long- term sequelae provided that intestinal length is sufficient.

How well did you know this?
1
Not at all
2
3
4
5
Perfectly
29
Q

A 27-year-old man with a long-standing history of Crohn’s disease is noted to have several extraintestinal manifestations of Crohn’s disease including erythema nodosum, arthritis, ankylosing spondylitis, anemia, and episodes of pancreatitis. During evaluation of his right lower quadrant pain, he is found to have a segment of thickened ileum causing obstruction. Which extraintestinal manifestations of his Crohn’s disease would you not expect to subside after resection of the involved segment of bowel?

A. Erythema nodosum

B. Arthritis

C. Ankylosing spondylitis

D. Anemia

E. Pancreatitis

A

ANSWER: C

COMMENTS: The extraintestinal manifestations of Crohn’s disease are listed in Table 22A.1.

They are not a primary indication for surgery in most patients with this disease.

Indications for surgery include obstruction, perforation, abscesses, internal or cutaneous fistulas, and perianal disease. However, if the involved bowel were resected, most extraintestinal manifestations would improve except for ankylosing spondylitis and hepatic changes.

Surgery is not held to be curative in Crohn’s disease since it is a systemic disorder.

Surgery is indicated when medical therapy fails or when the side effects of medications (such as steroids) are significant.

Bowel-conserving approaches are important since the majority of patients will require multiple surgeries in their lifetime.

Resection is limited to the offending segment(s). If adjacent areas of bowel are affected but are not the cause of perforation, obstruction, or fistula formation, they should be spared and managed medically.

Obstruction due to fibrotic stricturing is the most common indication for abdominal surgery in patients with Crohn’s disease.

Options for obstructed segments of the bowel include segmental resection and primary anastomosis or ostomy, stricturoplasty, and, rarely, bypass procedures.

Repeated wide resections of small bowel may lead to a short gut syndrome. Stricturoplasty is beneficial in patients with multiple short areas of narrowing separated by normal intestine.

Perforation occurs in 15%–20% of patients and usually results in the formation of a contained abscess, phlegmon, or an internal fistula to the bowel, bladder, or vagina.

Enterocutaneous fistulas rarely occur in patients without previous operation, but they are common after surgery.

Free perforations into the peritoneal cavity with peritonitis are rare.

Anemia is common, but frank hemorrhage is rare. It may occur if an ulcer erodes into a large blood vessel.

Anorectal disease occurs in 50% of patients with Crohn’s disease and may be the presenting problem in 5% of patients.

Perirectal abscesses and/or fistulas develop in up to 30% of patients with Crohn’s disease of the small bowel and are separate from the diseased segment of small bowel.

Patients with Crohn’s disease have an increased risk for the development of cancer in comparison with the general population, but the risk for colon cancer does not approach the level seen in patients with chronic ulcerative colitis.

This difference may be related to the segmental nature of Crohn’s disease involving a smaller proportion of the colon and the shorter average period between diagnosis and colectomy in Crohn’s disease.

The risk, however, is not considered high enough to warrant prophylactic resection.

However, colonoscopic surveillance is indicated. Most cases of small bowel cancer associated with Crohn’s disease occur in patients with long-standing disease and are most commonly found in a previously bypassed segment of bowel or at the site of a chronic small bowel stricture.

How well did you know this?
1
Not at all
2
3
4
5
Perfectly
30
Q

A patient with Crohn’s disease and obstructing chronic fibrotic small bowel strictures not responding to medical therapy is taken to the operating room. Appropriate surgical management includes:

A. Resecting the diseased segments with frozen section evaluation of the margins

B. Avoiding bowel resection for long strictures

C. Resecting bowel to palpably normal tissue

D. Resecting only the obviously obstructing segment and preserving as much bowel as possible

E. Performing no more than two stricturoplasties during a single operation

A

ANSWER: C

COMMENTS: Proximal and distal margins of resection are determined by gross visual examination and palpation to soft pliable tissue.

Frozen sections should not be routinely performed.

Resecting to areas without microscopic evidence of inflammation does not reduce recurrence rates or decrease complications such as leaks and may lead to unnecessary loss of small bowel length.

All areas of significant stricturing must be addressed during the operation since leaving any will often result in rapid recurrence of symptoms and the need for additional surgery.

Bowel resection for strictures is the most common abdominal procedure performed in Crohn’s disease; however, stricturoplasty is preferred for short obstructing segments since no normal bowel is sacrificed at the margins.

The length of a stricture acceptable for stricturoplasty is debatable.

Different surgeons have advocated for various types of stricturoplasty to manage longer segments. However, the commonly accepted approach is to perform the now standard Heineke– Mikulicz type of stricturoplasty for strictures up to 3 to 4 cm long.

There is no defined limit for the number of strictures that can be treated during one operation; some studies have shown up to 19 stricturoplasties done at once without increased morbidity.

Remarkably, there is a low leak rate, and the disease at these sites usually resolves.

How well did you know this?
1
Not at all
2
3
4
5
Perfectly
31
Q

Which of the following statements is true of the microscopic
appearance of Crohn’s disease?

A. The disease is confined to the mucosa and submucosa.

B. Identification of noncaseating granulomas is required for diagnosis.

C. Granulomas demonstrating caseation without acid-fast bacilli confirm the diagnosis.

D. Submucosal fibrosis occurs secondary to bacterial invasion.

E. Marked lymphangiectasia is a prominent microscopic feature.

A

ANSWER: E

COMMENTS: Several microscopic features characterize but are not specific for Crohn’s disease.

These features progress from an early to a late phase of involvement and can be described as a granulomatous fibrotic inflammation progressing through all layers of the bowel wall.

In the early phase, edema of the entire bowel wall is seen, accompanied by lymphangiectasia and hyperemia and an increased proportion of goblet cells in an otherwise normal mucosa.

In the intermediate phase, thickening is caused by fibrosis of the submucosal and subserosal areas of the bowel.

Focal mucosal ulcers become numerous, and in 60% of patients, sarcoid-like granulomas appear, particularly in the submucosa, subserosa, and regional lymph nodes.

These granulomas contain epithelioid giant cells, do not caseate, and do not contain acid-fast bacilli.

The absence of granulomas does not exclude the diagnosis of Crohn’s disease.

Lymphangiectasia remains visible throughout the interme- diate and late phases.
In the late phase, the dense fibrosis exceeds that expected from the simple healing of an inflammatory insult and produces a fixed stenosis and partial obstruction of the lumen.

The mucosa is denuded over wide areas, with occasional islands of intact mucosal cells (pseudopolyps).

Glands deep in the mucosa resemble those of the pyloric region and are termed aberrant pyloric glands or Brunner gland metaplasia. The ulcers can be deep, and progression through the bowel wall may result in abscess and/or fistula formation.

How well did you know this?
1
Not at all
2
3
4
5
Perfectly
32
Q

Diarrhea is one of the common clinical manifestations of Crohn’s disease. Which of the following statements is true regarding this manifestation?

A. Most patients experience intermittent bloody diarrhea.

B. Diarrhea is the result of segmental inflammation, leading to decreased small bowel absorption.

C. Decreased bile salt absorption in the diseased terminal ileum produces choleretic diarrhea.

D. Diarrhea is frequently described as mucus or pus like.

E. Bloody diarrhea almost always produces anemia.

A

ANSWER: C

COMMENTS: Only 10% of patients with Crohn’s disease are initially seen in an acute stage, usually with symptoms similar to those of appendicitis. In most patients, the onset is insidious, with intermittent pain or discomfort being the most frequent and sometimes the only symptom.

The pain is often precipitated by a dietary indiscretion.

With advanced disease, the pain may become associ- ated with signs and symptoms of partial obstruction.

Symptoms worsen with eating, and many patients resort to a liquid diet.

Constant, localized pain, especially if associated with a palpable mass, suggests the presence of a phlegmon, abscess, or enteric fistula.

Diarrhea is the next most frequent symptom, and, unlike diarrhea in patients with chronic ulcerative colitis, it rarely contains mucus, pus, or blood.

Diarrhea is the result of several factors. The inflamed segment of small bowel has a decreased capacity to absorb intestinal contents.

In addition, the obstruction produced by this involved segment alters the absorptive capacity of the proximal part of the bowel.

Decreased absorption of bile salts in the diseased terminal ileum leads to bile salt–induced damage to the absorptive cells of the colonic mucosa and produces choleretic diarrhea.

One-third of patients initially have a fever, and about 50% experience weight loss, weakness, and fatigue.

Although the diarrhea is usually not bloody, persistent occult loss of blood frequently produces iron deficiency anemia, which may be aggravated by the deficiency of vitamin B12, which is absorbed in the terminal ileum.

Hypoproteinemia occurs because of increased loss of protein from the inflamed bowel mucosa.

Vitamin and mineral deficiencies are the results of decreased ingestion, altered metabolism, and decreased absorption.

How well did you know this?
1
Not at all
2
3
4
5
Perfectly
33
Q

A 26-year-old woman with a history of Crohn’s disease is experiencing a flare of her disease. She is 6 weeks pregnant. Which of the following is true regarding the use of corticosteroids in patients with inflammatory bowel disease?

A. Corticosteroids are unsafe to use in pregnant patients with an acute flare of Crohn’s disease.

B. Corticosteroids effectively maintain remission of Crohn’s colitis and ulcerative colitis during pregnancy.

C. Corticosteroids used in enema (topical) form are not absorbed into the systemic circulation and therefore have no systemic side effects.

D. Therapy every other day is effective in pregnant patients during acute flares.

E. Intravenous corticosteroids and adrenocorticotropic hormone (ACTH) are equally effective in patients with acute severe ulcerative colitis that is refractory to oral treatment during pregnancy.

A

ANSWER: E

COMMENTS: The use of steroids in patients with an acute flare of Crohn’s colitis or ulcerative colitis during pregnancy has been shown to be not only effective but also safe for the mother and fetus. This is also true of sulfasalazine.

Corticosteroids have never been shown to maintain remission of Crohn’s colitis or ulcerative colitis, and prolonged use often results in major side effects. Sulfasalazine and other 5-acetylsalicylic acid (5-ASA) products including mesalamine and coated 5-ASA compounds are effective in maintaining remission only in patients with ulcerative colitis.

Topical steroids in foam or enema preparations may be absorbed in small amounts (10%–20%). Alternate-day dosing has not been effective in patients with inflammatory bowel disease.

Intravenous ACTH is preferred instead of intravenous hydrocortisone by some, but controversy still exists regarding whether ACTH is more effective, even for previously untreated ulcerative colitis. An ACTH dose of 40 to 60 units over an 8-h period appears to be as effective as 300 to 400 mg/day of hydrocortisone.

ACTH use has waned in the face of newer medical therapies.

The duration of steroid therapy varies depending on the severity of the disease, but it should always be tapered on an individual basis with the goal of discontinuation.

Although some patients (10%–15%) are kept on a low maintenance dose when complete elimination leads to flare-up, steroid therapy should not be continued as maintenance in patients who have achieved complete remission.

Failure to achieve remission after 2 months of administering more than 15 mg of prednisone may be an indication for surgical management.

How well did you know this?
1
Not at all
2
3
4
5
Perfectly
34
Q

During resection of the terminal ileum and ascending colon for Crohn’s disease, a 38-year-old man had 3 feet of small bowel removed. Six months later, he presents complaining of persistent diarrhea. Contrast studies and endoscopy are normal. The most likely etiology is:

A. Malabsorption of bile salts

B. Reactivation of Crohn’s disease

C. Gastric acid hypersecretion

D. Bacterial overgrowth

E. Partial bowel obstruction

A

ANSWER: A

COMMENTS: The most likely etiology of diarrhea in this patient is malabsorption of bile salts.

Bile salts and vitamin B12 are absorbed in the terminal ileum.

After resection of the terminal ileum, bile salt reabsorption may be compromised and it enters the colon in much higher concentrations than normal.

Bile salts irritate the colonic mucosa and interfere with absorption of fluid and electrolytes, leading to increased frequency and watery stools.

Treatment for bile salt–induced diarrhea is oral cholestyramine, a bile salt–binding resin.

Reactivation of Crohn’s disease is common following surgery.

Rates vary from 28% to 73% at 1 year and from 77% to 85% at 3 years after ileal resection.

The lowest rates of recurrence are in patients with disease limited to the colon after total colectomy (20%).

Endoscopic recurrence rates are higher than symptomatic recurrence rates. Most colonic and terminal ileal recurrences may be seen with colonoscopy.

Bacterial overgrowth is more frequently associated with a nonfunctioning portion of bowel that leads to stasis.

Examples include dilated proximal bowel above a stenotic obstruction, intes- tinal diverticula, and diversion or bypass surgery.

Bacteria consume vitamins and nutrients. Symptoms include nausea, vomiting, bloat- ing, and diarrhea.

The diagnosis is made with an abnormal d-xylose breath test.

Symptoms generally improve with antibiotics, although repeated courses or maintenance may be needed if the situation is not surgically corrected.

Gastric acid hypersecretion is associated with short gut syndrome.

Short gut syndrome will most likely occur in patients with less than 100 cm bowel and no ileocecal valve or in those with less than 50 cm bowel and an ileocecal valve.

This patient has had approximately 90 cm of bowel removed, leaving behind enough bowel that short gut syndrome is unlikely.

How well did you know this?
1
Not at all
2
3
4
5
Perfectly
35
Q

Nutritional support may be beneficial in patients with inflammatory bowel disease refractory to medical treatment. Which of the following statements is true?

A. Bowel rest and parenteral nutrition are the primary therapy for Crohn’s colitis.

B. In those with Crohn’s disease and a high-output fistula, total parenteral nutrition (TPN) promotes closure of the fistula.

C. TPN helps prevent the need for total colectomy in patients with ulcerative colitis.

D. In patients with Crohn’s ileitis, TPN is superior to enteral nutrition for providing an adequate caloric replacement.

E. An elemental diet is the primary therapy for exacerbation of Crohn’s disease.

A

ANSWER: B

COMMENTS: TPN has no role as a primary therapy for ulcerative colitis, but it may help maintain a satisfactory nutritional state during bowel rest.

TPN does not prevent the need for colectomy in refractory cases.

The role of TPN in patients with Crohn’s colitis is not well established, but in those with Crohn’s colitis and small bowel involvement, TPN may improve remission rates and promote fistula closure.

Elemental diets have been sporadically shown by some to be effective in inducing remission of active Crohn’s disease.

The patient’s tolerance may be poor, however, and the results are not superior to those obtained with corticosteroids and sulfasalazine.

Peripheral parenteral alimentation (PPN) rarely provides adequate caloric replacement and may induce venous sclerosis and phlebitis.

Enteral nutrition is preferable to TPN, when possible, since complications are much lower and nutritional balance is better.

Nutrition delivered through the GI tract appears to preserve normal GI function, benefit normal immune function, and decrease systemic inflammation.

How well did you know this?
1
Not at all
2
3
4
5
Perfectly
36
Q

A 30-year-old woman has a bowel obstruction secondary to Crohn’s disease. She has undergone multiple previous small bowel resections. At laparotomy, multiple strictures are noted throughout her bowel. Which of the following statements is true?

A. Stricturoplasty should be considered only for patients with an isolated stricture.

B. Segmental bowel resections are preferable to stricturo- plasty for the current laparotomy.

C. Restricture at the stricturoplasty site has been seen in less than 5% of patients.

D. Anastomotic leakage and fistula formation following stricturoplasty have been seen in 50% of cases.

E. Because residual disease is left behind, reoperation for Crohn’s disease is more likely with stricturoplasty than with bowel resection.

A

ANSWER: C

COMMENTS: Stricturoplasty for Crohn’s disease was first per- formed in 1981 by Emanuel Lee based on his experience with patients with intestinal tuberculosis in India. The procedure was popularized by Victor Fazio and his colleagues at the Cleveland Clinic. Experience since then has shown it to be a safe alternative to resection in properly selected patients.

Stricturoplasty should be considered in any patient who has had extensive previous resections of diseased bowel and in whom further resection might create short bowel syndrome (SBS) and in those with multiple separated fibrotic small bowel strictures. Many strictures can be treated safely by a single laparotomy.

The entire small bowel must be inspected to avoid overlooking strictures that are not obvious.

This can be accomplished by passing a long intestinal tube through one of the stricturoplasty sites.

The catheter is passed both proximally and distally through the entire length of small bowel.

The balloon on the catheter is inflated to 2 cm, and then the catheter is gradually withdrawn, identifying all significant strictures that are marked with a suture.

A longitudinal incision is made over the stricture and extended for 2 cm proximally and distally beyond the stricture.

A biopsy is performed to rule out neoplasia.

The enterotomy is then closed transversely with a single layer of interrupted absorbable sutures.

The site is marked with a metal clip. If a single stricture or several strictures close together are encountered at a patient’s first surgery, resection rather than stricturoplasty is preferable because it eliminates the diseased bowel and establishes the diagnosis.

Patients treated by stricturoplasty have been compared with patients treated by resection.

The need for reoperation at the original site is similar.

Postoperative complications are infrequent. At the Cleveland Clinic, anastomotic leakage, abscesses, or fistulas have occurred in 9% of patients treated by stricturoplasty.

Restricture at the stricturoplasty site occurred in only 2%.

How well did you know this?
1
Not at all
2
3
4
5
Perfectly
37
Q

A 54-year-old man is being assessed for colicky abdominal pain and occasional nonbilious emesis. He denies fevers and does not have leukocytosis. He has a history of melanoma that was resected from his arm 5 years earlier. His upper GI (UGI) radiograph is shown in Fig. 22A.1. What is the next best step in this patient’s management?

A. Barium enema with pneumatic decompression

B. Exploratory laparotomy and manual reduction

C. Exploratory laparotomy, manual reduction, and resection of the involved segment

D. Nasogastric tube placement, intravenous fluid, and a trial of nonoperative management

E. Exploratory laparotomy and intestinal bypass

A

ANSWER: C

COMMENTS: Intussusception is the telescoping of one portion of an intestinal segment onto the lumen of an adjacent segment (Fig. 22A.2).

It is commonly seen in children.

The lead point is commonly a Meckel’s diverticulum or ileal lymphoid hyperplasia.

It is the most frequent cause of pediatric bowel obstruction.

It is rare in adults, in whom it accounts for less than 5% of cases of bowel obstruction.

Most cases of intussusception in adults are caused by a specific lead point, whereas only 8%–20% are idiopathic.

Causes of intussusception in adults include inflammatory bowel disease, adhesions, Meckel’s diverticulum, neoplasms including cancers and polyps such as those in Peutz-Jeghers syndrome (PJS), and intestinal tubes.

A barium enema is useful for intussusception in children because it is diagnostic and therapeutic. Barium enemas and pneumatic decompression are often effective at reduction of the intussusception.

However, these approaches are not useful in adults.

Surgery is the mainstay for symptomatic adult intussusception.

A formal bowel resection with oncologic principles is warranted when malignancy is suspected, such as in this patient with a history of melanoma.

Melanoma is the most common nonabdominal malignancy to metastasize to the small intestine.

How well did you know this?
1
Not at all
2
3
4
5
Perfectly
38
Q

A 26-year-old man arrives at the emergency department with a complaint of recurrent, colicky, mid-abdominal pain. Physical examination reveals a palpable abdominal mass and several areas of increased pigmentation on his lips, palms, and soles. He states that his father had a colon polyp removed several years ago. Computed tomography (CT) of the abdomen was performed (Fig. 22A.3). PJS is suspected. Which of the following is true of PJS?

A. It is a sex-linked recessive familial disease characterized by intestinal polyposis and mucocutaneous hyperpigmentation.

B. Polyps are most frequently located in the jejunum and ileum but can also be found in the stomach, duodenum, colon, and rectum.

C. Surgical treatment includes resecting all bowel containing polyps.

D. Peutz-Jeghers polyps have a high malignant potential.

E. Peutz-Jeghers polyps are typically adenomatous.

A

ANSWER: B

COMMENTS: PJS is an autosomal dominant familial disease characterized by intestinal polyposis and mucocutaneous hyper- pigmentation.

The polyps are hamartomas that are most frequently located in the jejunum and ileum, but they can also be found in the stomach, duodenum, colon, and rectum.

It is generally believed that their malignant potential is fairly low.

PJS can cause intussusception or hemorrhage.

Up to one-third of patients initially have abdominal pain and a palpable mass.

Symptoms may be self-limited as the intussusception comes and goes.

The diagnosis of intestinal polyps may be made with UGI and small bowel follow-through contrast studies, pill endoscopy, or at the time of surgery.

Magnetic resonance (MR) enterography may prove to be useful as this modality becomes more refined.

Surgery is indicated for persistent obstruction, frequent recurrences, or bleeding.

Surgical treatment is limited to conservative resection of the polyps through enterotomies or limited resection.

Only larger or intussuscepting polyps need to be removed. Intraoperative total endoscopy is the most accurate method of polyp identification.

How well did you know this?
1
Not at all
2
3
4
5
Perfectly
39
Q

Which of the following statements is true concerning the causes of intestinal obstruction?

A. Among adults, 20% of intussusception cases are associated with a pathologic process, most commonly a tumor.

B. A leading cause of bowel obstruction is early postoperative adhesions.

C. In the United States, adhesions account for more than 50% of cases of small bowel obstruction.

D. Richter’s hernia cannot lead to complete obstruction.

E. Hernias are the leading cause of obstruction in the United States.

A

ANSWER: C

COMMENTS: Peritoneal adhesions account for more than 50% of cases of small bowel obstruction in the United States.

Obstruction immediately after abdominal operations, however, is uncommon, occurring among only 1% of patients in the 4 weeks after laparotomy.

Hernias of all types are second only to adhesions as the most frequent cause of obstruction.

External hernias such as inguinal or femoral hernias can present with symptoms of obstruction.

Femoral hernias are particularly prone to incarceration and bowel necrosis because of the small size of the hernia inlet.

An important consideration is Richter hernia. In this variant, only a portion of the bowel wall is incarcerated. Richter hernia most frequently occurs in association with femoral or inguinal hernia. Yet complete obstruction can occur if more than one-half to two-thirds of the bowel circumference is incarcerated.

Approximately 5% of cases of intussusception occur among adults.

Intussusception occurs when one segment of bowel telescopes into an adjacent segment.

The result is obstruction and ischemic injury to the intussuscepting segment.

Of adult cases of intussusception, 90% are associated with pathologic processes.

Tumors, benign and malignant, can act as a lead point causing the intussusception in more than 65% of cases among adults.

How well did you know this?
1
Not at all
2
3
4
5
Perfectly
40
Q

Which of the following is true concerning postoperative ileus (POI)?

A. The presence of peritonitis at the time of surgery delays return of normal function.

B. The use of metoclopramide hastens the return of motility.

C. Contrast radiographic studies have no role in differentiating early postoperative bowel obstruction from POI.

D. The judicious use of intravenous patient-controlled analgesia has no effect on the return of small bowel motor activity.

E. Alvimopan has not been shown to affect the return of small bowel motor activity.

A

ANSWER: A

COMMENTS: The term “ileus” reflects underlying alterations in motility of the gastrointestinal tract that lead to functional (not mechanical) obstruction.

Differentiating normal POI and the prolonged course of paralytic ileus is based primarily on the time since operation and the clinical circumstances.

Besides the location of the previous operation (upper abdominal, lower abdominal, pelvic), the nature of the previous operation and the findings may contribute.

Peritonitis or spillage of noxious material delays the return of normal bowel function.

Differentiating paralytic ileus from mechanical obstruction often is difficult. Abdominal radiographs will reveal gas in segments of both the small and large bowels in POI.

Occasionally, UGI contrast radiography or CT can be helpful in discriminating the cause.

Early postoperative obstruction is uncommon and is particularly rare for upper abdominal surgery. Most cases occur after operations on the colon, particularly abdominoperineal resection.

There has been little success in the use of active prokinetic agents, such as metoclopramide, to shorten recovery times after lower abdominal pro- cedures; however, alvimopan has been shown to significantly decrease the incidence of POI.

Alvimopan competitively binds to mu-opioid receptors on the bowel. If given prior to the use of narcotics, it reduces the motility-slowing effects of these commonly used medications.

Thus recovery after POI can take longer with the use of intravenous narcotics than with epidural pain control.

Enhanced recovery after surgery protocols commonly include the use of alvimopan starting before surgery, epidurals, nonnarcotic pain medications, and regional blocks for pain control and may demonstrate improvement in time-to-recovery of bowel function.

How well did you know this?
1
Not at all
2
3
4
5
Perfectly
41
Q

Which of the following is true regarding the initial treatment of patients with acute, complete small bowel obstruction?

A. Immediate surgery is warranted as soon as the diagnosis is made.

B. Nasogastric decompression should be used for as long as possible in patients with complete small bowel obstruction to allow resolution.

C. The presence of fever, tachycardia, localized pain, or leukocytosis suggests strangulation and warrants prompt surgery.

D. All patients with complete small bowel obstruction require blood and plasma for resuscitation.

E. If a small bowel resection must be performed, a stoma and mucous fistula are necessary because of the high risk of anastomotic failure.

A

ANSWER: C

COMMENTS: Timing an operation for a small bowel obstruction requires considerable clinical judgment. The duration of initial resuscitation must be balanced against the need to prevent gangrene by prompt intervention.

Severe intravascular volume depletion can occur as a result of fluid sequestration (as much as 6 L) in the lumen of the bowel and peritoneal cavity.

Sodium, chloride, and potassium depletion frequently accompanies bowel obstruction.

Blood loss is unusual unless strangulation is present.

Before induction of general anesthesia, fluid and electrolyte replacement should be instituted with isotonic saline solution to normalize the heart rate, blood pressure, and urine output.

Potassium repletion should begin once adequate urine output is established.

Surgery is delayed until the patient is stabilized.

Nasogastric decompression is an important component of supportive therapy; nausea and vomiting are controlled by this measure, and the risk for aspiration is reduced.

Swallowed air is evacuated, thus further limiting intestinal distention.

In patients with adhesive partial bowel obstruction and no signs of strangulation (i.e., fever, tachycardia, localized abdominal pain, or leukocytosis), a 24- to 48-h period of bowel rest and nasogastric decompression is warranted.

Most patients with a partial obstruction will resolve spontaneously with the above measures.

Delay in surgical intervention for a complete small bowel obstruction is not recommended (beyond 1 to 2 days) because the likelihood of strangulation and ischemia increases and is higher than with partial bowel obstruction.

There is no increase in the anastomotic leakage rate of small bowel anastomoses in urgent versus elective small bowel resections, provided that the segment of bowel used for the anastomosis is healthy and not overly distended.

Therefore a proximal stoma and mucous fistula are seldom necessary following uncomplicated small bowel resection for obstruction.

How well did you know this?
1
Not at all
2
3
4
5
Perfectly
42
Q

An 85-year-old woman has severe abdominal pain and distention. She is tachycardic, oliguric, and acidotic. Abdominal radiographs show pneumobilia and a mass
(Fig. 22A.4). What is the best surgical management for this patient during an exploratory laparotomy?

A. Resection of the mass

B. “Milking” the mass distally past the obstruction

C. Cholecystectomy, enterotomy, and removal of the mass

D. Enterotomy and removal of the mass

E. Hepaticojejunostomy

A

ANSWER: D

COMMENTS: This patient has gallstone ileus.

Gallstone ileus accounts for only 1% of all intestinal obstructions but is widely discussed.

It is caused by the passage of a large stone through a biliary– enteric fistula (commonly the duodenum) thus producing a bowel obstruction.

Thus the name “gallstone ileus” is a misnomer since it is a true obstruction and not an ileus.

The most common site of obstruction in patients with gallstone ileus is the terminal ileum because of the narrow lumen at the ileocecal junction.

The most common manifestations of gallstone ileus are nausea, vomiting, and abdominal pain.

About 50% of patients will have gallbladder-related symptoms.

Plain abdominal radiographs can reveal pneumobilia, dilated loops of small bowel, and a calcified stone outside the gallbladder.

Gallstone ileus is treated surgically. Obstruction is relieved by milking the stone in a retrograde fashion and removing it through a proximal enterotomy.

The segment of bowel at the site of impac- tion should be inspected for evidence of ischemia and necrosis.

If an ischemic compromise has occurred, the ischemic bowel should be resected.

Takedown of the biliary–enteric fistula and cholecystectomy may be done during the initial laparotomy.

However, in patients who are not able to tolerate a prolonged operation, this may be deferred.

Although many surgeons recommend cholecystectomy and fistula repair at some point, it is not clear that this is absolutely necessary, particularly in elderly or infirm patients.

How well did you know this?
1
Not at all
2
3
4
5
Perfectly
43
Q

A 4-year-old male presents with blood per rectum. Technetium-99 pertechnetate scintigraphy suggests a bleeding Meckel’s diverticulum. Which of the following is the appropriate treatment?

A. Ileal segmental resection with primary reanastomosis

B. Diverticulectomy

C. Medical management with proton pump inhibitor and octreotide

D. Angiography for embolization

E. Push enteroscopy for endoscopic treatment

A

ANSWER: A
COMMENTS: See Question 20.

How well did you know this?
1
Not at all
2
3
4
5
Perfectly
44
Q

A woman is undergoing an open incisional hernia repair through a previous cesarean section incision. During the operation, this structure (Fig. 22A.5) is noted about 60 cm from the ileocecal valve. What is true regarding this incidental finding?

A. It is a true diverticulum.

B. This lesion may be found in various anatomic forms in 50% of the population.

C. Pancreatic tissue is the most common ectopic tissue found in this diverticula.

D. Most complications occur in the elderly.

E. Diverticulitis is the most common complication

A

ANSWER: A

COMMENTS: A Meckel’s diverticulum, the most frequently encountered diverticulum involving the small intestine, occurs in 2%–4% of the general population. It is a true diverticulum containing all layers of the bowel wall and arises from the antimesenteric border of the ileum, 50 to 75 cm from the ileocecal valve.

The diverticulum is a result of abnormal regression of the embryonic vitelline duct.

Frequently, there is a persistent band of tissue extending from the tip of the diverticulum to the umbilicus.

The rule of 2’s states that it usually presents by the age of 2, is 2 feet from the ileocecal valve, occurs in 2% of the population, and may contain one of the 2 types of heterotopic mucosa.

The diverticulum may contain ectopic gastric mucosa capable of producing peptic ulceration and bleeding in the adjacent ileal mucosa.

This ectopic gastric mucosa can be visualized with 99mTc-labeled scans.

Gastric tissue is the most common ectopic tissue and is found in 50% of these lesions.

Pancreatic tissue is the next most common, although colonic mucosa has been found rarely.

Clinical problems are most often seen in the young pediatric population.

The most frequent complications are bleeding, intussusception, and obstruction. The latter is generally caused by vol- vulus or twisting around the persistent band.

The least common complication is diverticulitis, which is clinically manifested as lower abdominal pain and is usually thought to be appendicitis on presentation.

Therapy consists of diverticulectomy for uncomplicated diverticulitis, and segmental ileal resection for complicated diverticulitis or bleeding, since the ulcer is usually outside of the diverticulum.

Prophylactic diverticulectomy for an incidentally found Meckel’s diverticulum is still somewhat controversial.

Some clinicians recommend not removing the diverticulum when found incidentally unless there is evidence of ectopic gastric mucosa or the neck of the diverticulum is narrow.

The rate of complication from a Meckel’s diverticulum is about 6.4% over a lifetime and becomes smaller as a person ages.

Other clinicians argue that postoperative complications after prophylactic removal are rare, and they should be removed when found.

In an otherwise uncomplicated procedure, particularly in a younger person, prophylactic diverticulectomy is reasonable.

How well did you know this?
1
Not at all
2
3
4
5
Perfectly
45
Q

Which of the following statements is true regarding duodenal, jejunal, and ileal diverticula?

A. Duodenal diverticula are true diverticula.

B. Duodenal diverticula are often multiple, whereas jejunal diverticula are often solitary.

C. Asymptomatic duodenal diverticula should be resected to avoid potentially serious complications.

D. Duodenal diverticula usually cause symptoms and are found during specific workup.

E. Asymptomatic jejunal diverticula do not require therapy.

A

ANSWER: E

COMMENTS: Small bowel diverticula are not terribly uncommon and may be found in the duodenum in 3%–5% of individuals and in the jejunum and ileum in 1%–2% on radiographs, endoscopy, and autopsies.

Most duodenal, jejunal, and ileal diverticula are asymptomatic and are found incidentally.

Diverticula of the duode- num, jejunum, and ileum are most commonly false (pulsion) diverticula and lack muscularis propria.
Duodenal diverticula are usually solitary and project medially toward the head of the pancreas.

Although most are asymptomatic, 10% of patients may have nonspecific epigastric symptoms, such as pain and bloating. They also may bleed and perforate. In instances of contained perforation, the abscess should be drained.

Gastrojejunostomy is the operation most often applicable, although biliary decompression is occasionally necessary when the duct is compressed.

When there is bleeding without inflammation, diver- ticulectomy is indicated, either from a dorsal approach using the Kocher maneuver or via a duodenotomy.

Jejunal and ileal diverticula are often multiple and project from the mesenteric border of the bowel into the leaves of the mesentery.

This type of diverticulum is more common in the jejunum than in the ileum. Bacterial overgrowth may be treated with antibiotics.

Bleeding and perforation are treated with segmental resection.

Asymptomatic diverticula of the duodenum, jejunum, or ileum do not require therapy.

46
Q

What is the most common finding with small bowel tumors?

A. Hematemesis

B. Abdominal pain

C. Perforation

D. Intussusception

E. Anemia

A

ANSWER: B
COMMENTS: See Question 23.

47
Q

What is the most common symptomatic small bowel tumor?

A. Lipoma

B. Gastrointestinal stromal tumor (GIST)

C. Hamartoma

D. Hemangioma

E. Adenoma

A

ANSWER: B

COMMENTS: The most frequent symptom of small bowel tumors is abdominal pain.

Common benign lesions of the small intestine include gastrointestinal stromal tumors, adenoma, hemangioma, and lipoma.

The most common benign small bowel tumor is an adenoma.

The most common symptomatic small bowel tumors are GISTs.

Symptoms associated with small bowel tumors are often vague and nonspecific such as anorexia, dyspepsia, and abdominal pain.

Patients may also have signs of obstruction that are usually related to intussusception but may also be due to malignant narrowing.

Occult bleeding is also a common initial symptom.

When small bowel neoplasms are suspected, a barium small bowel follow-through study is indicated and is generally diagnostic.

CT or MR enterography may also be useful.

GISTs arise from the interstitial cells of Cajal. They express CD117 and CD34.

These tumors grow intramurally and can cause obstruction.

At times, these tumors can reach considerable size and outgrow their blood supply, which results in GI bleeding.

Mitotic rates higher than 5 per 50 high-power fields increase the risk for recurrence.

Lipomas are now included in the GIST category.

There are three types of small bowel adenomas: tubular adenomas, villous adenomas, and Brunner gland adenomas (usually in the duodenum).

Most of these lesions are asymptomatic and found incidentally at the time of autopsy.

Adenomas present with bleeding or obstruction.

Polypectomy may be performed in the duodenum if the tumor is histologically benign.

Jejunal and ileal lesions are treated with segmental resection.

Hamartomas of the small bowel are usually associated with PJS. Patients also have 1- to 2-mm pigmented lesions located in the circumoral region of the face, buccal mucosa, palms, soles, and perianal region.

The most common initial symptom is a colicky abdominal pain, usually as a result of intermittent intussusception.

Hemangiomas are malformations consisting of a submucosal proliferation of blood vessels.

They can occur anywhere along the GI tract, with the jejunum being the most common small bowel segment involved.

Bleeding is the most frequent symptom. Angiography and 99mTc-labeled red blood cell scanning are useful diagnostic studies.

When identified, small bowel tumors should be excised because of the risk for complications, to establish the diagnosis, and to exclude cancer.

Small lesions can be excised with an enterotomy and primary closure.

Larger or suspicious lesions should be treated with segmental resection with primary anastomosis.

48
Q

What is the most common primary malignant small bowel tumor?

A. Leiomyosarcoma

B. Carcinoid

C. Lymphoma

D. GIST

E. Adenocarcinoma

A

ANSWER: E
COMMENTS: See Question 25.

49
Q

What is the most common finding in patients with malignant small bowel tumors?

A. Hematemesis

B. Perforation

C. Abdominal pain

D. Intussusception

E. Anemia

A

ANSWER: C

COMMENTS: Malignant tumors of the small bowel account for 2% of all GI malignancies. The most frequent primary type is adenocarcinoma, followed in decreasing frequency by carcinoid, GIST, and lymphoma.

Although adenocarcinoma occurs with equal frequency in the duodenum, jejunum, and ileum, the other types tend to occur most often in the ileum.

In contrast to benign lesions, malignant lesions of the small intestine are usually accompanied by pain and weight loss. Other clinical manifestations may include diarrhea, obstruction, or chronic blood loss with anemia.

Obstruction from malignant lesions is usually because of tumor infiltration, stricturing, and adhesion.

The preferred therapy is wide resection with regional lymphadenectomy.

For each entity, survival is dependent on a number of factors and is variable, but in general, GISTs have a 5-year survival rate that ranges from 7% to 56%, lymphomas have a 5-year survival rate of about 40%, and adenocarcinoma has the lowest survival, at about 20%.

Postoperative chemotherapy and radiation therapy can be useful in treating a patient with lymphoma but are not useful adjuncts for adenocarcinoma or sarcoma.

Histiocytic lymphoma may develop in patients with long-standing celiac sprue and has a worse prognosis than do conventional small bowel lymphomas.

The Mediterranean-type lymphoma, a variant associated with monoclonal alpha heavy chains and a dense plasma cell tumor infiltration, also carries a poor prognosis.

50
Q

A 35-year-old man presents with obstructive symptoms and, on exploration, is found to have a mid-jejunal mass. Frozen section reveals a GIST. Which of the following is true regarding GISTs?

A. GISTs originate from the interstitial cells of Cajal.

B. The most frequent site of GISTs is the small bowel.

C. Nodal status is the most reliable predictor of aggressive behavior.

D. Postoperative imatinib mesylate is used in all patients to reduce recurrence.

E. On endoscopy, a GIST typically appears as an ulcerating mucosal lesion.

A

ANSWER: A

COMMENTS: GISTs are the most common nonepithelial neoplasms of the small bowel.

They originate from the interstitial cells of Cajal.

GISTs are most common in the stomach (60%), followed by the small bowel (30%).

On endoscopy, GISTs classically appear as submucosal lesions.

The majority express the CD117 antigen of the KIT transmembrane receptor tyrosine kinase, which is the product of the KIT protooncogene.

Mutation in KIT leads to abnormally activated KIT protein and enables oncogenic signaling in the cell. This discovery led to the development of systemic therapy with imatinib mesylate (Gleevec), which blocks KIT signaling and inhibits tumor proliferation.

Postoperative imatinib has been shown to prolong recurrence-free survival and is considered standard fol- lowing resection of GIST tumors larger than 3 cm.

The clinical behavior of GISTs is highly variable, but GISTs > 1 cm have increased malignant potential.

Tumor size and mitotic rate are the two major criteria used to define aggressive behavior.

51
Q

A 54-year-old man reports with a 2-month history of abdominal pain and significant weight loss. Upper endoscopy, colonoscopy, and CT were all normal. A barium upper GI study with small bowel follow-through identified a mass in his mid-ileum. At surgical exploration, a carcinoid tumor, confirmed by frozen section, was found in the mid-ileum. Which statement is true regarding his condition?

A. Prognosis is primarily related to lymph node status.

B. The cell of origin is the Kulchitsky cell.

C. The ileum is the most common site of origin.

D. A 1-cm distal ileum carcinoid tumor should be treated with a formal right hemicolectomy.

E. Resection is not indicated in patients with metastatic disease.

A

ANSWER: B

COMMENTS: The origin of carcinoid tumors is the Kulchitsky cell, which is thought to arise from the neural crest.

Carcinoids can occur anywhere in the GI tract. The most frequent site is the appendix, followed by the ileum and rectum.

Extraintestinal sites include the bronchus and ovaries.

Small bowel carcinoid tumors tend to be multiple in 30% of cases, and a second GI tumor of another histologic type can be found in 30% of patients.

The prognosis is a function of the size of the tumor and its site of origin.

Ileal carcinoids tend to metastasize more commonly than those that originate in the appendix.

The usual submucosal location of carcinoid tumors often makes them difficult to find on radiographic examination or with cursory palpation during an exploratory laparotomy.

The tumors may incite an intense fibrotic reaction in the surrounding soft tissue and mesentery, which can cause luminal narrowing.

Mesenteric lymph node and liver metastases can be large in comparison with the primary tumor.

Tumors less than 1 cm in diameter and without demonstrable metastases can be treated by excision or segmental resection.

Those larger than 1 cm or with regional metastases should be excised widely along with lymph node clearance; right hemicolectomy is indicated for >1-cm lesions of the distal ileum and appendix.

For patients with metastases (local or distant) and in whom carcinoid syndrome is present, removal of the primary tumor and debulking of metastatic disease can provide considerable palliation.

52
Q

Which of the following is true of carcinoid syndrome?

A. Cardiac manifestations commonly affect the mitral and aortic valves.

B. 5-Hydroxyindoleacetic acid (5-HIAA) is the active metabolite leading to carcinoid syndrome symptoms.

C. Diarrhea affects less than 30% of patients.

D. The most useful diagnostic test for suspected carcinoid syndrome is the determination of serum serotonin levels.

E. Functional carcinoid tumors divert dietary tryptophan into the production of serotonin.

A

ANSWER: E

COMMENTS: Episodic manifestations of carcinoid syndrome include flushing, diarrhea, and asthma.

The cutaneous manifestations are the most common and consist of episodes of flushing of the face, neck, arms, and upper part of the trunk, occasionally accompanied by vasomotor collapse.

Diarrhea is significant in more than 80% of patients and is usually sudden in onset, watery, and accompanied by cramping pain and borborygmi.

Asthmatic attacks occur in 25% of patients.

Manifestations of long-standing involvement include the development of facial hyperemia with telangiectasias on the cheeks, nose, and forehead; development of the cutaneous lesions of pella- gra; and valvular heart disease.

The valves most commonly involved are the tricuspid and pulmonic, although the mitral and aortic valves are sometimes affected.

Peripheral edema is present in about 70% of patients and can occur in the absence of valvular disease.

Functioning carcinoid tumors divert up to 60% of dietary tryptophan into the production of serotonin, thereby contributing to the development of pellagra and protein deficiency.

Serotonin is metabolized in the liver to 5-HIAA, which is excreted in urine. For this reason, the most useful diagnostic test in patients suspected of having a carcinoid tumor is the determination of 5-HIAA levels in a 24-h collection of urine.

5-HIAA is inactive and does not cause carcinoid syndrome.

Carcinoid syndrome is produced by release of serotonin into the systemic circulation either by liver metastases or by tumors located outside the portal distribution.

Although it is generally believed that patients with carcinoid syndrome have tumors that produce serotonin, the role of serotonin in the mediation of the syndrome is not clear.

Not all patients with elevated production of serotonin have the syndrome.

Some patients with the syndrome have normal levels of 5-HIAA in urine, and injection of pure serotonin does not create all of the manifestations of the disease.

It is likely that carcinoid tumors have the capacity to produce a number of biologically active peptides, which accounts for the variability of the syndrome and discrepancies between a patient’s serotonin levels and the clinical findings.

Other substances produced by carcinoid tumors include histamine, dopamine, kallikrein, substance P, prostaglandins, and neuropeptide K.

Treatment of carcinoid crisis includes intravenous octreotide, intravenous antihistamine, and hydrocortisone.

53
Q

During a routine appendectomy, a 1-cm mass is found at the tip of the appendix. Frozen section is concerning for carcinoid tumor. What is the best treatment option for this patient?

A. Right hemicolectomy

B. Medical therapy with octreotide

C. Neoadjuvant therapy with streptozotocin and 5-fluorouracil

D. Appendectomy

E. Ileocecectomy

A

ANSWER: D
COMMENTS: See Question 30.

54
Q

On abdominal exploration for a suspected carcinoid tumor, a 2-cm mass is found in the terminal ileum. No liver lesions were detected on preoperative imaging or with intraoperative palpation. What is the best treatment option for this patient?

A. Appendectomy

B. Right hemicolectomy

C. Medical therapy with octreotide

D. Neoadjuvant therapy with streptozotocin and 5-fluorouracil

E. Ileocecectomy

A

ANSWER: B

COMMENTS: Treatment of small bowel carcinoid is based on tumor size and the presence or absence of metastatic disease.

Segmental resection is adequate for tumors smaller than 1 cm without regional lymph node metastasis.

Wide excision is indicated for lesions larger than 1 cm, multiple tumors, or regional lymph node metastasis.

Right hemicolectomy is indicated for lesions of the terminal ileum or base of the appendix.

Debulking is indicated for metastatic carcinoid tumors. This may involve liver resection.

Hepatic artery ligation or percutaneous embolization has also produced good results in controlling the carcinoid symptoms produced by liver metastasis.

Medical therapy for patients with malignant carcinoid is directed at relieving symptoms.

Octreotide, a somatostatin analogue, helps relieve symptoms in most patients.

Regression of tumor with the use of octreotide has been reported.

Interferon-alfa has also been shown to relieve symptoms.

Chemotherapeutic agents such as streptozotocin and 5-fluorouracil have had limited success in treating malignant carcinoid.

They are used mostly in patients with metastatic disease who are symptomatic and unresponsive to other therapies.

55
Q

Somatostatin has emerged as a safe and effective agent with a broad range of applications. Which of the following is true about the use of somatostatin in patients with carcinoid tumors?

A. Somatostatin may be used as a provocative agent before measuring 5-HIAA levels.

B. Somatostatin receptor scintigraphy is more effective than CT or magnetic resonance imaging (MRI) in localizing primary and metastatic carcinoid tumors.

C. Somatostatin is ineffective for the management of carcinoid crisis.

D. Somatostatin therapy improves survival in patients with carcinoid syndrome.

E. Administration of somatostatin can be used as a provoca- tive diagnostic test.

A

ANSWER: B

COMMENTS: Somatostatin was first identified in 1973. Since then, a great deal of interest has been directed at characterizing and identifying its physiologic effects and the clinical utility of somatostatin and its analogues.

Somatostatin is a 14–amino acid protein with several analogues of shorter lengths that maintain clinical effectiveness. The general effects of somatostatin are those of an inhibitory hormone. Several provocative agents may be used before conducting tests for neuroendocrine tumors, including pentagastrin, secretin, and calcium infusion. Somatostatin is not effective as a provocative agent.

Somatostatin receptor scintigraphy uses indium-111 and a gamma camera.

This study has several advantages over conven- tional imaging (CT or MRI).

Its sensitivity is higher (90% vs. 70%) for metastatic disease, it is more effective in identifying the primary tumor site, and it visualizes the entire body to detect occult metastases.

Carcinoid tumors visible by somatostatin receptor scintigraphy suggest that these particular tumors have somatostatin receptors and are therefore subject to the inhibitory effects of somatostatin.

Carcinoid crisis is a life-threatening event that may develop during episodes of flushing, anesthesia, or surgery.

Severe hypotension and bronchospasm may occur during carcinoid crises, and they may be refractory to the usual supportive care.

The reported incidence of such crises is variable and ranges from 2% to 50%.

Somatostatin may be administered preoperatively as a prophylactic agent or during a carcinoid crisis as a therapeutic agent.

It is usually successful in reversing the condition.

Somatostatin has also been found to be highly effective in relieving the symptoms of carcinoid syndrome.

It has been suggested that chronic octreotide therapy results in longer survival in patients with carcinoid syndrome than in those treated with chemo- therapy, but this hypothesis remains to be proven by randomized, controlled trials.

56
Q

A 62-year-old woman complains of abdominal pain and weight loss. She undergoes a small bowel follow-through study (Fig. 22A.6). Her past medical and surgical history includes a stable small right lung nodule, removal of a skin lesion on her right leg, and an endoscopic polypectomy of a gastric polyp. She has a strong family history of breast cancer. If the lesion noted in the stomach is metastatic, what is the most likely primary cancer?

A. Squamous cell skin cancer

B. Lymphoma

C. Lung cancer

D. Breast cancer

E. Melanoma

A

ANSWER: E

COMMENTS: Metastatic tumors of the small bowel are more common than primary tumors. The most common metastases to the small bowel are primary tumors arising from other intra abdominal organs.

In these cases, small bowel involvement occurs by either direct extension or implantation of tumor cells.

Extra abdominal metastasis to the small bowel is rare. Cutaneous melanoma is the most common extra abdominal source of metastasis to the small bowel.

Other extra abdominal sources include breast and lung cancers.

Treatment is palliative resection or bypass if the metastatic tumor is not amenable to resection.

57
Q

A 56-year-old woman had a right hemicolectomy for villous adenoma of the cecum. Five days after surgery her surgical wound becomes red and tender. She underwent pelvic radiation therapy 5 years ago for cervical cancer. The surgeon opens her wound with immediate drainage of purulent fluid. The drainage persists as a continuous brown liquid discharge over the next day. Which of the following is the most likely diagnosis?

A. Simple wound infection

B. Clostridial infection

C. Anastomotic leakage with an enterocutaneous fistula

D. Dehiscence

E. Cellulitis

A

ANSWER: C

COMMENTS: Most fistulas are iatrogenic and result from anas- tomotic leakage, inadvertent injury to the bowel during the opera- tion, laceration of the bowel during an abdominal closure, or retained foreign bodies.

Less than 2% of fistulas are the result of diseased bowel.

When they are, the most common contributing factors are preoperative radiation therapy, intestinal obstruction, and inflammatory bowel disease.

Although small bowel fistulas occasionally lead to generalized peritonitis, they most commonly produce a walled-off abscess manifested as an infection of the operative incision.

The initial drainage may be purulent, but if the infection is caused by anastomotic leakage of the small bowel, the drainage becomes enteric within 1 to 2 days.

58
Q

For the patient described in Question 33, which of the following is the most appropriate initial management?

A. Packing of the subcutaneous tissue with wet-to-dry dressings

B. Packing of subcutaneous tissue with dry, absorbent dressings

C. Immediate return to the operating room for exploration

D. Skin protection and appliance placement to collect the drainage

E. Sutured closure of the fascial defect

A

ANSWER: D

COMMENTS: The initial management of a small bowel fistula includes the administration of appropriate intravenous fluids, proximal decompression with nasogastric suction, control and quantification of the output of the fistula, and protection of the surrounding skin.

Fistulas are classified according to their locations and the volumes of their output.

Proximal fistulas tend to have higher output and lead to more severe electrolyte and fluid imbalances.

Nasogastric suction can be helpful in diminishing the output of proximal intestinal fistulas, but the output of those more distal in the gut may not be influenced by this maneuver.

Sump catheters can provide a means of controlling and quantifying high-output fistulas, especially early in their formation.

Maintaining proper position of the catheter in the wound can be problematic.

Once the fistula tract is established, suction catheters should be promptly replaced with a stoma appliance fixed to the edges of the fistula.

Enteric contents are highly corrosive, and the skin surrounding the fistula opening should be protected carefully.

Gauze dressings are generally ineffective at absorbing all the drainage and protecting the skin.

Therefore their use is generally avoided.

Most well- established fistulas do not produce sepsis, but in patients with persistent fever, systemic administration of antibiotics and a careful search for an undrained abdominal abscess are indicated.

Early in the workup of this patient and before the GI tract has been filled with contrast material (from conventional GI radio- graphs), CT should be performed to look for areas of abscess for- mation or fluid accumulation.

It may also identify the site of the fistula. If CT does not show the site, fistulography is helpful. If one is concerned about distal obstruction, a small bowel follow-through study may provide information, if this issue was not satisfactorily answered by CT or fistulography.

59
Q

Diagnostic workup of the woman described in Question 34 reveals that she has a distal ileal fistula that is communicat- ing with a small cavity. Daily output of the fistula is 100 mL. Which of the following is appropriate therapy?

A. Prompt exploration and division of the fistula tract

B. Prompt exploration and bypass of the fistula

C. Prompt exploration and resection of the involved ileum with primary anastomosis

D. Six weeks of intravenous hyperalimentation

E. Two weeks of low-residue or elemental enteral alimentation

A

ANSWER: E

COMMENTS: Knowing the location of the fistula is of important prognostic and therapeutic value.

The overall mortality rate for small bowel fistulas is 20%; the rate is higher for jejunal fistulas and lower for those of the ileum.

With proper supportive care, such as intravenous or enteral alimentation, and in the absence of distal obstruction, up to 40% of small bowel fistulas close spontaneously.

Enteral alimentation has the advantage of avoiding the possible hepatic and septic complications associated with pro- longed TPN.

Even if there is a slight increase in fistula output after the start of enteral nutrition, the fistula may still close.

Fistulas of the proximal jejunum may require transnasal insertion of a long tube through the stomach and duodenum and just beyond the fistula before starting enteral alimentation. Surgery should be avoided for 4 to 6 weeks to permit spontaneous closure and allow the local inflammation to subside, thereby facilitating subsequent surgery.

The preferred operation for correcting a persistent fistula is resection of the fistula in continuity with the segment of involved bowel, followed by primary anastomosis.

Alternative therapies include complete or partial exclusion with primary anastomosis.

60
Q

One year after undergoing antrectomy and Billroth II reconstruction for peptic ulcer disease, a patient is being evaluated for anemia. The patient is also noted to have vague epigastric pain that is relieved by projectile bilious emesis without food particles. Which statement is true regarding this patient’s condition?

A. Medical management with tetracycline and vitamin B12 can definitively correct the condition.

B. Bacteria successfully compete for vitamin B6, which may lead to megaloblastic anemia.

C. Bacterial deconjugation of bile salts can lead to steatorrhea.

D. The addition of intrinsic factor in the Schilling test causes urinary vitamin B12 excretion to return to normal.

E. The addition of tetracycline in the Schilling test causes urinary vitamin B6 excretion to return to normal.

A

ANSWER: C

COMMENTS: The blind loop syndrome is caused by stasis of the intestinal contents with subsequent bacterial overgrowth.

This stasis can be caused by a number of abnormalities, including stricture, stenosis, fistula, diverticulum, or the formation of a blind pouch (as noted in a Billroth II operation).

Steatorrhea, diarrhea, anemia, weight loss, abdominal pain, multiple vitamin deficiencies, joint pains, and occasionally neurologic disorders characterize the syndrome.

The steatorrhea is the result of bile salt deconjugation in the stagnant fluid in the blind loop of bowel.

Megaloblastic anemia is probably a result of successful competition by bacteria for vitamin B12.

The Schilling test reveals a type of urinary excretion of vitamin B12 similar to that seen with pernicious anemia except that it is corrected, not by the addition of intrinsic factor, but by the use of oral tetracycline.

Although the administration of tetracycline and parenteral vitamin B12 can correct megaloblastic anemia, only surgical correction of the cause of the bowel stasis is curative.

Surgical correction includes converting the Billroth II to a Billroth I operation or the creation of Roux-en-Y limb gastrojejunostomy with a vagotomy to prevent marginal ulceration.

61
Q

Which of the following statements regarding SBS is true?

A. Resection of up to 70% of the bowel can be tolerated if the terminal ileum and ileocecal valve are preserved.

B. Diarrhea is best controlled by the administration of medium-chain triglycerides.

C. The administration of oral bile salts is of central importance in controlling steatorrhea.

D. Vagotomy and pyloroplasty and reversal of a segment of bowel are the two most important operations for the early management of SBS.

E. Relative gastric hyposecretion, with increased intestinal pH in conjunction with interruption of the enterohepatic bile salt circulation, is the cause of steatorrhea.

A

ANSWER: A

COMMENTS: SBS may be diagnosed when there is significant malabsorption of vitamins and minerals or an inability to maintain caloric enteral nutrition in the setting of extensive small bowel resection.

The usual quoted rule that less than 100 cm of intact small bowel results in SBS is not terribly useful since SBS may develop in a variety of situations and amounts of residual bowel.

The normal length of small bowel from the ligament of Treitz to the ileocecal valve is about 500 cm, but a wide range was found, from 250 to 850 cm, in a recent study.

The entire jejunum can be resected without adverse nutritional sequelae.

The entire ileum can be resected without harm as long as vitamin B12 is administered regularly afterward.

Up to 70% of the small bowel can be resected safely if the terminal ileum and ileocecal valve are left intact.

If they are resected, however, loss of 50%–60% of the small bowel can lead to severely compromised nutrition.

The deficiencies created by extensive resection of the small bowel are vitamin B12 malabsorption (terminal ileum), altered fat absorption, and fluid and electrolyte problems.

Vitamin B12 malabsorption leads to vitamin B12 deficiency and megaloblastic anemia.

Altered fat absorption produces steatorrhea as a result of several factors.

First, massive small bowel resection leads to gastric hypersecretion because decreased bowel pH stimulates the intestine, thereby shortening transit time and interfering with the absorption of ingested fat.

Second, interruption of bile salt resorption interferes with micelle formation.

Third, the unabsorbed fats are irritating to the colonic mucosa, thereby increasing diarrhea and steatorrhea associated with the syndrome.

Fluid and electrolyte problems are a function of the shortened transit time and diarrhea that results from loss of small bowel absorptive area.

Treatment of SBS centers on control of diarrhea and parenteral maintenance of nutrition.

With time (2 to 3 years), the mucosa of as little as 30 to 45 cm of small bowel may undergo enough hypertrophy to allow withdrawal of intravenous alimentation and the start of carefully modified oral feedings.

Treatment with growth hormone, glutamine, and fiber has shown some promise in stimulating gut regeneration.

Diarrhea can be controlled with agents such as Lomotil or codeine that slow intestinal motility.

Oral calcium carbonate is also useful and acts by neutralizing hydrochloric acid and free fatty acids.

When oral intake is resumed, dietary fat is restricted to 30 to 50 g daily.

Some patients benefit from the use of medium-chain triglycerides.

Oral bile salts are tolerated and aid in the formation of micelles in some patients, whereas in others, they cause increased diarrhea.

Cholestyramine, an agent that sequesters bile acids, is useful in patients who have had less than 100 cm of small bowel resected.

There is no standard approach to the resumption of oral intake, and treatment must be highly individualized.

Although some patients ultimately do well with a modified oral diet, others remain dependent on permanent parenteral nutrition.

There are no operative procedures that reliably correct SBS.

Therefore operative treatment should be considered only in patients who cannot maintain their body weight within 30% of normal without intravenous supplementation.

Operations that may be useful are reversal of a segment of intestine, creation of a recirculating loop of small bowel, creation of an artificial sphincter, vagotomy and pyloroplasty, correction of bowel obstruction, and placing all bowel in continuity (i.e., reversal of preexisting stomas).

Vagotomy and pyloroplasty have rarely been performed for SBS since the introduction of H2 blockers and proton pump inhibitors.

Allotransplantation of small bowel in humans has been performed successfully, but has a high failure rate and is only available in a few specialized centers.

62
Q

During a small bowel resection, you have finished your anastomosis and have just closed the mesenteric defect. As you are closing the midline wound, you see a large hematoma in the area of the mesenteric defect. Choose the best treatment option.

A. Observation alone

B. If hematoma is expanding, open, explore, and obtain hemostasis

C. Resect the involved segment and redo the anastomosis

D. Proceed with closure if the hematoma is expanding but the anastomosis appears perfused

E. Evacuate the hematoma under all circumstances

A

ANSWER: B

COMMENTS: The hematoma should be observed for several minutes to evaluate if it is expanding.

If there is no evidence of expansion or ongoing bleeding, one can proceed with closing.

Not every hematoma requires intervention.

If the hematoma is large or expanding, the mesentery should be opened and explored.

Hematomas can expand and tamponade the veins, leading to arterial inflow obstruction and poor blood flow to the anastomosis.

Only if you cannot adequately control the hematoma should the involved segment be resected and the anastomosis redone.

63
Q

Peyer’s patches are primarily responsible for the local
synthesis of:

A. IgD
B. IgE
C. IgG
D. IgM 
E. IgA
A

ANSWER: E

COMMENTS: Nodules of lymphoid tissues found throughout the small intestine and colon are known as Peyer’s patches.

IgA is elaborated by plasma cells and is unique in that this immunoglobulin is transcytosed directly across enterocytes or hepatocytes for secretion directly into the intestinal contents or bile.

Peyer’s patches are part of the mucosal-associated lymphoid tissue (MALT).

64
Q

Which of the following statements regarding tuberculous enteritis is incorrect?

A. Primary infection usually results from the ingestion of nonpasteurized milk contaminated with Mycobacterium bovis.

B. Secondary infection results from the ingestion of bacilli contained in contaminated sputum.

C. The duodenum is the site of involvement in 85% of patients.

D. Infection may be indistinguishable from Crohn’s disease or cancer.

E. Approximately one-half of patients with colonic or ileocolonic disease may be treated medically without surgery.

A

ANSWER: C

COMMENTS: Primary enteral tuberculosis is rare in the United States, but is still common in underdeveloped countries where ingestion of unpasteurized milk occurs more commonly.

Usually, it causes minimal symptoms, but occasionally, it results in stricturing and stenosis in the ileocecal area.

The radiographic findings may be indistinguishable from those of carcinoma of the colon.

Although it may be necessary to resect bowel because of high-grade obstruction, it is not appropriate to do so simply to establish the diagnosis.

This can be accomplished with biopsy alone.

Treatment with isoniazid, p-aminosalicylic acid, and streptomycin usually suffices.

Ulcerative tuberculosis is a form that develops secondary to pulmonary disease and is more common than the primary form of this disease in the United States.

Symptoms are nonspecific and variable, but most often consist of pain and diarrhea.

Suspicion should be raised in Asian patients, particularly those coming from the Indian subcontinent.

The most common laboratory findings are anemia and an elevated C-reactive protein.

The diagnosis can be strongly suggested by barium examination, CT with intravenous (IV) and per os (po) contrast, or MR enterography.

Confirmation is obtained by a positive tuberculin skin test, although a negative test does not rule out the diagnosis.

Not infrequently, the diagnosis is made by open or laparoscopic exploration and biopsies.

An appropriate response to antitubercular therapy also confirms the diagnosis.

Surgery is best reserved for complications such as perforation, obstruction, or hemorrhage.

65
Q

Which of the following statements regarding typhoid enteritis is true?

A. Culturing Salmonella typhi from blood or stool can make the diagnosis.

B. Chloramphenicol is the preferred treatment.

C. Bleeding requiring operative intervention occurs in 10%–20% of patients.

D. Steroids have no use in treating typhoid enteritis.

E. Hyperplasia and ulceration of Peyer’s patches and
mesenteric lymphadenopathy are rare findings.

A

ANSWER: A

COMMENTS: Typhoid enteritis, a systemic infection caused by S. typhi, is accompanied by fever, headache, coughing, maculopapular rash, abdominal pain, and leukopenia.

Hyperplasia and ulceration of Peyer’s patches, mesenteric lymphadenopathy, and splenomegaly also occur.

Chloramphenicol is not the drug of choice because of the emergence of resistant strains of bacteria and the risk for marrow toxicity.

Currently, trimethoprim-sulfamethoxazole (Bactrim) is preferred.

Patients who remain in a toxic state after 1 week of therapy often benefit from a short course of prednisone.

Bleeding occurs in 10%–20% of patients and is usually treated by transfusion. Perforation through ulcerated Peyer’s patches occurs in 2% of patients and is most often free, solitary, and located in the terminal ileum.

Operative closure and appropriate peritoneal toilet are required.

Occasionally, the perforations are multiple, which necessitates intestinal resection with primary anastomosis.

66
Q

Favorable factors affecting spontaneous fistula closure rates?

A
Continuity maintained
End fistula
No associated abscess
Healthy adjacent bowel
Free flow distally
Duodenal stump
Jejunal
Tract >2cm
Defect <1cm
Patients with optimal nutritional status
67
Q

Nonfavorable factors affecting spontaneous fistula closure rates?

A
Compete disruption
Lateral fistula
Associated abscess
Diseased adjacent bowel
Distal obstruction
Lateral duodenal
Ileal
Tract <2cm
Defect >1cm
Patients with poor nutritional status
68
Q

Factors that inhibit spontaneous closure of fistulas?

A
Foreign body within tract
Radiation enteritis
Infection/inflammation
Epithelialization of the tract
Neoplasm at the fistula origin
Distal obstruction of the intestine
69
Q

Rates of spontaneous closure for fistulas?

A

> 90% in 1 month
<10% after 2 months
None after 3 months

70
Q

Most favorable time to reoperate on fistulas?

A

Either within 10 days of diagnosis or after 4 months

71
Q

Epidemiology of small bowel neoplasms?

A

<5% of primary GI malignancies arise from small intestines

1/3 of primary small bowel neoplasms are benign
2/3 are malignant

Less common in women (M>F)

6th-7th decade of life

Malignant more common in distal small bowel (except adenoCA— duodenal predilection)

72
Q

Management of duodenal adenomas?

A

If <2cm: endoscopic polypectomy

If >2cm: transduodenal polypectomy or segmental duodenal resection, or pancreaticoduodenectomy

73
Q

Management of duodenal adenomas in patients with FAP?

A
  • Endoscopic polypectomy

- Surveillance endoscopy at 6 month intervals, then annually

74
Q

Management of duodenal adenocarcinoma?

A
  • Pancreaticoduodenectomy for proximal lesions (right of superior mesenteric artery)
  • Segmental resection for distal duodenal lesions (left of SMA)
75
Q

Management of jejunal or ileal tumors?

A

Segmental resection with 5cm of tumor free proximal and distal margins

76
Q

Management of localized small bowel carcinoid?

A

Wide en bloc resection (include adjacent mesenteric and LN)

77
Q

Management of metastatic carcinoids?

A

Debulking surgery

78
Q

Management of carcinoid syndrome?

A
  • Preoperative somatostatin or octreotide
  • Debulking including resection of hepatic mets
  • Loperamide, diphenoxylate, or cyproheptadine (serotonin receptors antagonist) for the diarrhea
  • Type 1 and 2 histamine receptor antagonists for flushing
  • Albuterol and aminophylline for bronchospasm and wheezing.
79
Q

Management of localized or resectable GIST?

A

Wide local excision of primary tumor with in continuity resection of adherent organs

80
Q

Management of Unresectable or metastatic GIST?

A

Imatinib (Gleevec)

81
Q

Management of advanced disease in small bowel malignancies?

A

Palliative resection or bypass (diversion)

82
Q

Management of small bowel lymphoma?

A
  • Primary chemotherapy (CHOP regimen)

- Segmental resection for tumor complications

83
Q
  1. A male neonate develops small-bowel obstruction due to malrotation of the midgut segment. An x-ray of the abdomen confirms the presence of small-bowel obstruction (Fig. 6–1). He undergoes an emergency laparotomy, untwisting of the malrotated intestines, and partial small-bowel resection for intestinal infarction. Which of the following statements is true of the small intestine (jejunum and ileum)?

(A) It is derived entirely from the midgut.

(B) In the fetus, it enters the physiologic umbilical hernia in the the fifth month.

(C) It remains in the physiologic hernia for 4 months.

(D) It is attached to the urachus.

(E) It drains into the lymph nodes around the iliac arteries.

A
  1. (A) The small intestine arises from the midgut
    segment.

The midgut segment extends between the ampulla of Vater and the distal transverse colon.

It enters the physiological umbilical hernia at sixth week and returns to the peritoneal cavity by the tenth week.

The vitellointestinal tract (site from which Meckel’s diverticulum arises) is attached to the antimesenteric margin of the distal ileum. The urachus is attached to the bladder.

The intestinal lymphatic drainage is directed to the preaortic glands.

84
Q
  1. A 64-year-old man with mitral stenosis develops mesenteric infarction due to an embolus. At operation and on subsequent pathologic examination, what is noted regarding the small intestine (jejunum and ileum)?

(A) It commences at the right of the midline.

(B) It contains crypts but not villi on histologic examination.

(C) It has a mesentery (parietal) attachment extending 61 cm along the posterior abdominal wall.

(D) It measures approximately 6m in length.

(E) It is supplied by the inferior mesenteric vessels.

A
  1. (D) The small intestine commences to the left of
    the midline at Treitz’s ligament and ends at the ileocecal junction.

The mesenteric attachment is only 15 cm in length.

It is supplied by the midgut vessel (superior mesenteric).

The sympathetic and parasympathetic (vagus) nerves enter the mesentery to supply the vessels and gut wall.

85
Q

A 64-year-old woman with a known history of cardiac disease is admitted to the hospital with severe abdominal pain. Her blood pressure is 150/95 mmHg, and her pulse rate is 84 beats per minute (bpm). There are minimal signs of intravascular depletion.

  1. The possibility of small-bowel infarction is characterized by which of the following?

(A) The stack-of-coins sign

(B) Marked distention of loops of bowel

(C) Air in the biliary tree

(D) Air in the bowel wall (intramural)

(E) Air below the left diaphragm

A
  1. (D) Gangrene of the bowel occurs before the ominous sign of intramural air can be detected.

The stack-of-coins sign is seen in intestinal obstruction where the proximal small intestine folds are stacked to provide this characteristic feature on a plain x-ray of the abdomen.

86
Q

A 64-year-old woman with a known history of cardiac disease is admitted to the hospital with severe abdominal pain. Her blood pressure is 150/95 mmHg, and her pulse rate is 84 beats per minute (bpm). There are minimal signs of intravascular depletion.

  1. At operation, 2.5 m of distal ileum is found to be gangrenous. There is, however, pulsation in the superior mesenteric artery and its main branches. Small-bowel gangrene in this patient is caused by which of the following?

(A) Arterial thrombosis

(B) Embolus

(C) Nonocclusive ischemic disease

(D) Von Willebrand’s disease

(E) Idiopathic thrombocytopenic purpura

A
  1. (C) In a patient with small intestine infarction,
    the possibility of nonocclusive ischemic disease should be excluded by angiography.

If there is no evidence of gangrene, then fluid resuscitation and intra-arterial superior mesenteric papaverine administration may be adequate, and surgical intervention may be avoided.

Von Willebrand’s disease is characterized by a mild to moderate fall in factor VIII levels (pseudohemophilia) but with a much milder bleeding tendency than in true hemophilia.

It affects males and females equally.

87
Q
  1. A 48-year-old man undergoes a supine abdominal x-ray for epigastric discomfort. He has been on IV hyperalimentation since an operative procedure performed 5 days previously. Gas is consistently absent from the alimentary tract because he has previously undergone which of the following?

(A) Appendectomy

(B) Gastrostomy

(C) Ligation of the esophagus and cervical esophagostomy

(D) Lysis of adhesions

(E) Colostomy for large-bowel obstruction

A
  1. (C) Most air that reaches the stomach and intestines comes from swallowed air.

Air is nearly always seen in the small intestine on a plain film of the abdomen.

Gas in the stomach is derived mainly from swallowed air, which has an oxygen content of 20% and nitrogen content of 80%.

CO2 is formed by organic fermentation and comprises 40% of the gases in the distal bowel.

Nitrogen is absorbed so that it is reduced below 50% distally.

Methane and hydrogen sulfide gases are added in the distal bowel.

88
Q

A 64-year-old woman is admitted to the hospital with abdominal pain, vomiting, and abdominal distention. Bowel sounds are increased on auscultation, and a plain film shows marked distention of loops of bowel with nonspecific pattern.

  1. The most likely diagnosis is which of the following?

(A) Sigmoid volvulus

(B) Cecal volvulus

(C) Jejunal obstruction

(D) Ileal obstruction

(E) Pyloric obstruction

A
  1. (D) A plain film of the abdomen shows valvulae conniventes in jejunal (proximal) obstruction, a featureless bowel pattern in distal ileal obstruction, and haustra in colon obstruction.
89
Q

A 64-year-old woman is admitted to the hospital with abdominal pain, vomiting, and abdominal distention. Bowel sounds are increased on auscultation, and a plain film shows marked distention of loops of bowel with nonspecific pattern.

  1. Management, following rehydration and electrolyte imbalance correction, should initially
    involve which of the following?

(A) Nasogastric suction, rehydration, and observation

(B) Anticholinergic drugs

(C) Laxatives

(D) Emergency surgery and bowel resection

(E) Appendectomy

A
  1. (A) The initial management of intestinal obstruction is to correct fluid and electrolyte imbalance.

Surgery is indicated if strangulation is anticipated or if the obstruction fails to respond to conservative management.

Nasogastric suction is often effective in obstruction because of adhesions but is contraindicated when the obstruction is caused by a hernia and/or strangulation is suspected.

90
Q
  1. A 42-year-old woman is admitted to the emergency department with severe colicky pain, vomiting, and abdominal distention. She has not passed stools or flatus for 48 hours. X-rays of the abdomen confirm the presence of small bowel obstruction. What is the most likely cause of small-bowel obstruction in this patient?

(A) Adenocarcinoma

(B) Adhesions

(C) Crohn’s disease

(D) Ulcerative colitis

(E) Gallstone ileus

A
  1. (B) In patients presenting with small-bowel obstruction, clinical examination can usually identify a groin swelling attributable to strangulated hernia.

If external groin hernia is excluded, the presence of an abdominal scar would highly suggest that intestinal obstruction is caused by adhesions.

Peritoneal metastasis and primary tumors, bands, Crohn’s disease, and gallstone ileus must be excluded.

The distention is mainly a result of swallowed air.

If the obstruction is proximal, the onset is usually more severe and rapid.

91
Q
  1. An 80-year-old woman with a known history of femoral hernia is admitted to the hospital because of strangulation of the hernia. There is a tender swelling in the right femoral region immediately below and lateral to the pubic tubercle. She has had multiple bowel movements without relief of symptoms. What is the most likely diagnosis?

(A) Lymphadenitis

(B) Diverticulitis

(C) Volvulus

(D) Richter’s hernia

(E) Gastroenteritis

A
  1. (D) In a Richter hernia, only part of the circumference of the bowel wall has become trapped in the hernia sac, and normal bowel movements may still occur.

In the presence of a reducible groin hernia, it is important on clinic examination to be certain that other pathologic conditions are not overlooked.

92
Q
  1. A 63-year-old man from Miami presents to the emergency department with abdominal pain due to intestinal obstruction. A diagnosis of small-bowel volvulus is established. Primary small-bowel volvulus is differentiated from secondary small bowel volvulus. In the latter there is a secondary cause, such as adhesions, that accounts for the volvulus. Which is true of primary small-bowel volvulus?

(A) It does not lead to gangrene of bowel.

(B) It is common in the United States.

(C) It occurs nearly exclusively in women.

(D) It usually involves the jejunum.

(E) It may require a limited resection of small intestine.

A
  1. (E) Primary small-bowel volvulus is common in
    countries where the diet is high in bulk.

Except for the neonatal variety (associated with malrotation), it is rare in the United States.

Small-bowel volvulus secondary to adhesions is more common here.

The ileum is more frequently involved than the jejunum.

If a small-bowel resection is required, it is usually of a limited nature.

93
Q

A 44-year-old man is stabbed in the abdomen. The
injury penetrates the root of the small-bowel mesentery. At laparotomy, resection of 2 cm of ileum is removed.

  1. The complication that is more likely to occur after resection of the ileum rather than of an equivalent length of jejunum is the failure to absorb which of the following?

(A) Iron

(B) Zinc

(C) Bile salts

(D) Medium-chain triglycerides

(E) Amylase

A
  1. (C) The ileum is the exclusive site of bile salt
    absorption, and failure of its absorption contributes to the steatorrhea.

Ileal resection, which at times includes the ileocecal valve, is more commonly performed than is proximal resection.

Over a longer period of time (2–3 years), megaloblastic anemia occurs.

94
Q

A 44-year-old man is stabbed in the abdomen. The
injury penetrates the root of the small-bowel mesentery. At laparotomy, resection of 2 cm of ileum is removed.

  1. Why is distal resection, as compared to proximal resection, poorly tolerated?

(A) Transit time in the ileum is slower than that in the jejunum.

(B) Transit time in the jejunum is slower than that in the ileum.

(C) The greater bulk of food is absorbed in the ileum.

(D) Water absorption is mainly in the ileum.

(E) All minerals are absorbed preferentially in the ileum.

A
  1. (A) Transit time in the ileum is slower than that
    in the jejunum.

Resection of equal lengths of intestine results in greater deterioration after ileal resection as the site of slower (and therefore more complete) absorption is removed.

Jejunal resection is followed by hypertrophy of the residual villi in the ileum and functional compensation to a degree greater than in the jejunum after ileal resection.

95
Q
  1. A 66-year-old woman is admitted for hyperalimentation due to malnutrition consequent to massive small-bowel resection. What is the most likely condition that leads to the need to perform a massive resection?

(A) Autoimmune disease

(B) Mesenteric ischemia

(C) Mesenteric adenitis

(D) Cancer

(E) Pseudomyxoma peritonei

A
  1. (B) Massive resection occurs if more than
    75–80% is resected (leaving less than 1 m of
    small bowel).

The most common indications for major bowel resection are ischemia, Crohn’s disease, volvulus, and trauma.

96
Q
  1. A 68-year-old female is known to have had surgery several years previously for a bowel lesion. Her surgeon had told her that she suffers from the blind loop syndrome. In which condition can one anticipate the blind loop syndrome to occur?

(A) Intestinal bypass

(B) Vesicocolic fistula

(C) Duodenal ulcer disease

(D) Multiple polyposis of the colon

(E) Anteriovenous fistula of the colon

A
  1. (A) In the blind loop syndrome, bacteria proliferate in an affected segment that fails to show appropriate peristaltic activity.

It may be seen in surgery requiring jejunal or ileal bypass, small intestinal diverticular disease, scleroderma, diabetes mellitus, and intestinal carcinoma.

Macrocytic anemia, caused by malabsorption of Vitamin B12 and folic acid, is a key diagnostic feature in its diagnosis.

97
Q
  1. A 33-year-old woman is noted to have a Meckel’s diverticulum when she undergoes an emergency appendectomy. The diverticulum is approximately 60 cm from the ileocecal valve and measures 2–3 cm in length. What is the most common complication of Meckel’s diverticulum among adults?

(A) Bleeding

(B) Perforation

(C) Intestinal obstruction

(D) Ulceration

(E) Carcinoma

A
  1. (C) Intestinal obstruction due to a Meckel’s diverticulum may result from a volvulus, band obstruction, or intussusception.

Among children, bleeding and inflammation are seen more frequently.

Meckel’s diverticulum is a remnant of the vitellointestinal duct.

98
Q
  1. A 30-year-old male is diagnosed with Peutz-Jeghers syndrome. What findings is consistent
    with the diagnosis?

(A) Adenomas

(B) Hamartomas

(C) Adenomatous polyps

(D) Villoglandular polyps

(E) Villotubular polyps

A
  1. (B) Peutz–Jeghers syndrome is rare but should be considered if pigmented spots are found on the lips, mouth, or hands.

Hamartomas are not neoplasms; the name is derived from the Greek hamartos, which refers to the misfiring of a javelin.

The tissues appropriate to the site misfire and are arranged in an irregular order.

99
Q
  1. A 38-year-old male is admitted to hospital with symptoms suggestive of small-bowel obstruction. Examination reveals multiple loops of distended bowel with increased bowel sounds. Treatment with IV fluids and nasogastric suction fails to correct symptoms. Laparotomy is performed. Following surgery, copious volumes of fluid occur through the incision. A diagnosis of intestinal fistula is established. What is TRUE of intestinal fistulas?

(A) They may occur as a complication after an operation to divide adhesions.

(B) They are rare after irradiation.

(C) As a result of Crohn’s disease, they almost always close spontaneously.

(D) They should not be treated with a central venous line for fear of sepsis.

(E) They most commonly arise from the distal colon.

A
  1. (A) Unfortunately, in most series, division of adhesions accounts for as much as 25% of postoperative intestinal fistulas.

These cases usually involve sites that are not recognized at the time of operation.

The fistulas occurring after resection of the bowel in Crohn’s disease are less likely to heal without surgical intervention.

The small intestine is the most common site of intestinal fistula formation.

100
Q
  1. A 69-year-old female is found to have an enterocutaneous fistula that arises from the proximal small intestine. Which of the following statements is TRUE concerning this fistula?

(A) If internal, it occurs mainly from iatrogenic causes.

(B) It occurs more commonly after an anastomosis than spontaneously.

(C) If internal, it always causes serious complications.

(D) If external, it closes spontaneously in 10% of cases.

(E) If external, it requires immediate closure in most cases.

A
  1. (B) Internal small-bowel fistulas are caused
    almost exclusively by small-bowel disease or surrounding visceral disease involving the small
    bowel.

Crohn’s disease is the most common cause of internal small-bowel fistulas, but neoplasia, lymphoma, and tuberculosis must be excluded.

Internal fistula may be asymptomatic or cause serious malabsorption (proximal to distal fistulas) or infection (enterovesical fistulas).

101
Q

A 40-year-old woman experiences flushing, diarrhea, and wheezing. On physical examination, she is found to have tricuspid valve insufficiency.

  1. What is the most likely diagnosis?

(A) Appendiceal carcinoid

(B) Ileal carcinoid with liver metastasis

(C) Gastric lymphoma

(D) Small-bowel adenocarcinoma

(E) Bronchial carcinoid

A
  1. (B) The carcinoid syndrome in patients with intestinal carcinoid tumors will occur only in the presence of hepatic metastasis.

Approximately 40% of patients with hepatic metastasis from an ileal carcinoid will develop the syndrome.

102
Q

A 40-year-old woman experiences flushing, diarrhea, and wheezing. On physical examination, she is found to have tricuspid valve insufficiency.

  1. The most useful diagnostic finding is which of the following?

(A) Elevated 5-hydroxyindoleacetic acid (5-HIAA) levels

(B) Elevated blood sugar levels

(C) Elevated serum gastrin levels

(D) Elevated amylase levels

(E) Elevated norepinephrine levels

A
  1. (A) Patients with carcinoid tumor due to ovarian dermoid or pulmonary lesion may develop the syndrome with an elevated 5-HIAA, although hepatic metastasis are absent.

The liver does not counteract the hormone in this instance, because the portal system is bypassed.

103
Q
  1. A 56-year-old man has suffered from intermittent claudication for 5 years. He has recently developed cramping abdominal pain that is made worse by eating. He has a history of a 15-lb weight loss. What is the most likely diagnosis?

(A) Chronic intestinal ischemia (intestinal angina)

(B) Chronic cholecystitis

(C) Esophageal diverticulum

(D) Peptic ulcer

(E) Abdominal aortic aneurysm

A
  1. (A) Patients with underlying ischemic disease may develop acute intestinal infarction or intestinal angina, which is aggravated by eating.
104
Q
  1. A 68-year-old male musician presents to the emergency department with a sudden onset of colicky abdominal pain and massive vomiting of 4-hour duration. Examination shows an elevated WBC of 13,200 with a HCT of 45%. Electrolytes and blood urea nitrogen (BUN) are normal. An erect film of the abdomen reveals dilatation of the stomach with distended loops of bowel. What is his clinical diagnosis?

(A) Complete proximal intestinal obstruction

(B) Incomplete proximal intestinal obstruction

(C) Complete ileal obstruction

(D) Incomplete ileal obstruction

(E) Small-bowel perforation

A
  1. (A) Mechanical obstruction implies a barrier that impedes progress of intestinal contents.

Complete mid- or distal small-bowel obstruction presents with colicky abdominal pain, more marked
abdominal distention but with vomiting that is less frequent and occurs at a later stage than that of proximal jejunal obstruction.

105
Q
  1. What is true with reference to small-bowel physiology migrating motor complexes (MMC)?

(A) They are increased after feeding.

(B) They occur once every 10 minutes.

(C) They continue throughout laparotomy.

(D) They inhibit nutrient absorption.

(E) They may explain diarrhea that occurs following vagotomy.

A
  1. (E) MMC are isoperistaltic waves and occur approximately once every 90 minutes.

Oral feeding inhibits the MMC for as much as 3–4 hours.

The inhibition of the MMC in the stomach and intestine may account in part for nausea and vomiting occurring after surgery.

The major force that drives chyme aborally is that of segmentation and not the MMC.

106
Q
  1. A 38-year-old man with a history of fever associated with abdominal pain of 3-week duration presents now with a sudden onset of abdominal pain and copious vomiting. Plain abdominal x-rays reveal air under a diaphragm. A CT scan shows mesenteric lymphadenopathy and splenomegaly is found. Laparotomy is performed and 3 feet of ileum resected. The luminal aspect of the resected bowel shows marked ulceration of Peyer’s patches. What is the most likely diagnosis?

(A) Typhoid enteritis

(B) Tuberculosis enteritis

(C) Crohn’s disease

(D) Primary peritonitis

(E) Ulcerative colitis

A
  1. (A) Typhoid fever typically presents with initial
    symptoms.

Small intestine complications are related to involvement of Peyer’s patches of the small intestine, which result in bleeding and/or perforation in the second and third week after symptoms are noted.

107
Q
  1. A 48-year-old man is admitted to hospital because of a 3-day history of mild abdominal pain, repeated vomiting, and marked abdominal distension. Immediately after the pain commenced, he had one small-bowel movement but no further passage of stool or flatus. An abdominal flat plate revealed marked distension of loops of bowel confined to the small bowel. A plain abdominal film shows loops of bowel that all extensively show valvulae conniventes. What is the most likely site of obstruction (Fig. 6–3)?

(A) High-small bowel

(B) Mid-small bowel

(C) Rectum

(D) Colon

(E) Duodenum

A
  1. (B) The absence of loops of colon makes a colonic site most unlikely as a cause of the current clinical presentation. Distention does not occur in high small-bowel obstruction.
108
Q
  1. Following insertion of a nasogastric tube and appropriate rehydration and electrolyte correction, there is no change in clinical presentation. What should the next step involve?

(A) Barium reduction with controlled hydrostatic pressure

(B) Laparoscopy

(C) Colostomy

(D) Needle tap to deflate bowel

(E) Exploratory laparotomy

A
  1. (E) In view of the presence of bowel obstruction, surgery is indicated.

In general, patients who have obstruction due to adhesions may undergo an initial short trial period of conservative management.

Laparotomy is usually indicated in bowel obstruction due to other causes, where gangrene may be evident, and in all cases in which an initial period of conservative treatment fails.

109
Q

A 44-year-old female immigrant from India, and now resident in the US, has been treated for partial intestinal obstruction due to tuberculosis. There is no evidence of intestinal perforation.

  1. What should the next step in treatment involve?
    (A) Laparoscopy

(B) Laparotomy and bowel resection

(C) A full course of antituberculous drugs

(D) Steroids

(E) Radiation therapy to the abdomen

A
  1. (C) Tuberculosis is the great mimicker of disease and, therefore, should always be considered in the differential diagnosis of different abdominal conditions.

Surgical intervention will be required if the obstruction becomes complete.

110
Q

A 44-year-old female immigrant from India, and now resident in the US, has been treated for partial intestinal obstruction due to tuberculosis. There is no evidence of intestinal perforation.

  1. What is the most likely outcome for the patient?

(A) Full recovery

(B) Rapid deterioration and possible death

(C) Pneumonia

(D) Empyema

(E) Scrofula

A
  1. (A) Although intestinal tuberculosis still remains relatively uncommon in the United States, it should be particularly excluded in the AIDS population.

In these patients, the rarity of the condition may make its clinical detection particularly difficult.

Always suspect tuberculosis in the differential diagnosis of fever without a clearly defined cause.

111
Q
  1. A 64-year-old woman presents with a strangulated femoral hernia. At operation, what is the criterion used to determine the viability of a loop of bowel?
(A) Increased peristalsis
(B) Absent arterial pulsation
(C) Venous engorgement
(D) Intraoperative CT scan
(E) Serum amylase
A
  1. (B) The blood supply to a loop of ischemic bowel is determined by the presence or absence of arterial pulsation, peristalsis, and color of the bowel
    after resuscitation and relief of obstruction.